Pharmacology Exam 3

¡Supera tus tareas y exámenes ahora con Quizwiz!

A patient with metastatic cancer has had several fractures secondary to bone metastases. The provider orders denosumab [Xgeva]. What will the nurse teach this patient? a. Denosumab may delay healing of these fractures. b. Denosumab should be given subcutaneously every 12 months. c. Denosumab will improve hypocalcemia. d. Unlike bisphosphonates, denosumab does not increase osteonecrosis of the jaw (ONJ).

ANS: A Because denosumab suppresses bone turnover, fracture healing may be delayed. Denosumab is given every 6 months. Denosumab can exacerbate hypocalcemia. Denosumab can increase the incidence of ONJ.

The nurse is caring for a patient receiving intravenous acyclovir [Zovirax]. To prevent nephrotoxicity associated with intravenous acyclovir, the nurse will: a. hydrate the patient during the infusion and for 2 hours after the infusion. b. increase the patient's intake of foods rich in vitamin C. c. monitor urinary output every 30 minutes. d. provide a low-protein diet for 1 day before and 2 days after the acyclovir infusion.

ANS: A The nurse should ensure that the patient is hydrated during the acyclovir infusion and for 2 hours after the infusion to prevent nephrotoxicity. Increasing vitamin C would not help prevent nephrotoxicity. Monitoring urine output is important but would not help prevent nephrotoxicity. A low-protein diet is not indicated after an acyclovir infusion.

A patient comes to the clinic and receives valacyclovir [Valtrex] for a herpes-zoster virus. The nurse instructs the patient to take the medication: a. without regard to meals. b. without any dairy products. c. each morning. d. on an empty stomach.

ANS: A The patient may take the medication without regard to meals. The patient does not need to avoid dairy products, take the pill only in the morning, or take it on an empty stomach.

A nurse is providing education about tetracycline [Sumycin]. Which statement by the patient best demonstrates understanding of the administration of this medication? a. "I should not take this medication with milk or other dairy products." b. "I should not worry if I experience an acnelike rash with this medication." c. "I should take an antacid, such as Tums, if I experience gastrointestinal distress." d. "I should take this antibiotic with a calcium supplement to improve absorption."

ANS: A The patient should avoid taking the medication with dairy products to help prevent chelation. An acnelike reaction would indicate an allergic response. Taking the medication with calcium-containing antacids or supplements should be avoided, because this also leads to chelation.

A patient is receiving tobramycin three times daily. A tobramycin peak level is 4.5 and the trough is 1.2. What will the nurse do? a. Give the next dose as ordered. b. Hold the next dose and notify the provider. c. Monitor the patient for signs of nephrotoxicity. d. Tell the patient to report tinnitus.

ANS: A These levels are within normal limits, so the next dose may be given safely. It is not necessary to withhold the next dose. These levels do not indicate any increased risk of nephrotoxicity or ototoxicity.

A patient received 500 mg of azithromycin [Zithromax] at 0800 as a first dose. What are the usual amount and time of the second dose of azithromycin? a. 250 mg at 2000 the same day b. 500 mg at 2000 the same day c. 250 mg at 0800 the next day d. 500 mg at 0800 the next day

ANS: C Azithromycin generally is given as 500 mg on the first day and then 250 mg/day for the next 4 days, so the second dose would be 24 hours after the first dose.

1. A pregnant patient is treated with trimethoprim/sulfamethoxazole (TMP/SMZ) [Bactrim] for a urinary tract infection at 32 weeks' gestation. A week later, the woman delivers her infant prematurely. The nurse will expect to monitor the infant for: a. birth defects. b. hypoglycemia. c. rash. d. kernicterus.

ANS: D Kernicterus is a disorder in newborns caused by deposition of bilirubin in the brain. Sulfonamides promote kernicterus by displacing bilirubin from plasma proteins. Sulfonamides should not be given to infants under 2 months of age or to pregnant women after 32 weeks' gestation. Sulfonamides do not cause birth defects or hypoglycemia. Serious rashes may occur but are not the primary concern in the newborn.

A provider orders intravenous moxifloxacin [Avelox] for a patient who has sinusitis. Before administering the drug, the nurse will review this patient's chart for: a. a history of asthma. b. concurrent use of digoxin. c. concurrent use of warfarin. d. recent serum electrolyte levels.

ANS: D Moxifloxacin prolongs the QT interval and poses a risk of serious dysrhythmias. Patients with hypokalemia have an increased risk, so serum electrolyte levels should be monitored. Having a history of asthma is not significant. Moxifloxacin does not alter digoxin or warfarin levels.

A nurse is preparing to administer oral ofloxacin to a patient. While taking the patient's medication history, the nurse learns that the patient takes warfarin and theophylline. The correct action by the nurse is to request an order to: a. reduce the dose of ofloxacin. b. increase the dose of ofloxacin. c. increase the dose of theophylline. d. monitor coagulation levels.

ANS: D Ofloxacin increases plasma levels of warfarin, so coagulation tests should be monitored. The ofloxacin dose should not be reduced or increased. Ofloxacin does not affect theophylline levels.

A female patient who has hepatitis C is being treated with pegylated interferon alfa and ribavirin [Ribasphere]. It will be important for the nurse to teach this patient that: a. if she gets pregnant, she should use the inhaled form of ribavirin [Virazole]. b. if she is taking oral contraceptives, she should also take a protease inhibitor. c. she should use a hormonal contraceptive to avoid pregnancy. d. she will need a monthly pregnancy test during her treatment.

ANS: D Ribavirin causes severe fetal injury and is contraindicated during pregnancy. Women taking ribavirin must rule out pregnancy before starting the drug, monthly during treatment, and monthly for 6 months after stopping treatment. Inhaled ribavirin is also embryo lethal and teratogenic. Adding a protease inhibitor will reduce the efficacy of oral contraceptives. Women using ribavirin should use two reliable forms of birth control.

A nurse is obtaining a drug history from a patient about to receive sulfadiazine. The nurse learns that the patient takes warfarin, glipizide, and a thiazide diuretic. Based on this assessment, the nurse will expect the provider to: a. change the antibiotic to TMP/SMZ. b. increase the dose of the glipizide. c. monitor the patient's electrolytes closely. d. monitor the patient's coagulation levels.

ANS: D Sulfonamides interact with several drugs and through metabolism-related interactions can intensify the effects of warfarin. Patients taking both should be monitored closely for bleeding tendencies. Changing to the combination product will not help, because sulfonamides are still present. Sulfonamides intensify glipizide levels, so this drug may actually need to be reduced. Trimethoprim, not sulfonamides, raises potassium levels.

A patient has had three gouty flare-ups in the past year. Which drug class will the nurse expect the provider to order for this patient? a. Colchicine b. Glucocorticoids c. Nonsteroidal anti-inflammatory drugs d. Urate-lowering drugs

ANS: D The provider will order a urate-lowering drug for this patient. The medication should be diluted and administered with 20 mL of sterile sodium chloride and administered over 5 minutes or longer.

A patient who has been receiving intravenous gentamicin for several days reports having had a headache for 2 days. The nurse will request an order to: a. discontinue the gentamicin. b. obtain a gentamicin trough before the next dose is given c. give an analgesic to control headache discomfort. d. obtain renal function tests to evaluate for potential nephrotoxicity.

ANS: A A persistent headache may be a sign of developing ototoxicity, and since ototoxicity is largely irreversible, gentamicin should be withdrawn at the first sign of developing ototoxicity. A gentamicin trough should be obtained before the next dose is given when high gentamicin levels are suspects. Analgesics are not indicated until a serious cause of the headache has been ruled out. A headache is an early sign of ototoxicity, not nephrotoxicity.

A 65-year-old patient who receives glucocorticoids for arthritis is admitted to the hospital for treatment of a urinary tract infection. The prescriber has ordered intravenous ciprofloxacin [Cipro]. Before administering the third dose of this drug, the nurse reviews the bacterial culture report and notes that the causative organism is Escherichia coli. The bacterial sensitivity report is pending. The patient complains of right ankle pain. What will the nurse do? a. Withhold the dose of ciprofloxacin and notify the provider of the patient's symptoms. b. Instruct the patient to exercise the right foot and ankle to minimize the pain. c. Question the patient about the consumption of milk and any other dairy products. d. Request an order to increase this patient's dose of glucocorticoids.

ANS: A A rare but serious adverse effect associated with fluoroquinolones is tendon rupture, and those at highest risk are children, patients older than 60 years, transplant patients, and any patients taking glucocorticoids. Any pain in either heel should be reported and the drug should be discontinued. Patients should be instructed not to exercise until tendonitis has been ruled out. Dairy products can reduce the absorption of ciprofloxacin, so this is not a concern with this patient. Because the pain may be caused by tendonitis associated with ciprofloxacin, it is not correct to request an increase in the glucocorticoid dosing.

A patient with a urinary tract infection is given a prescription for TMP/SMZ. When reviewing the drug with the patient, the nurse learns that the patient has type 1 diabetes mellitus and consumes alcohol heavily. What will the nurse do? a. Contact the provider to request a different antibiotic for this patient. b. Obtain frequent blood glucose determinations while giving TMP/SMZ. c. Suggest that the patient take a potassium supplement while taking TMP/SMZ. d. Tell the patient to avoid excessive fluid intake while taking TMP/SMZ.

ANS: A Alcoholics are likely to be folate deficient and have an increased risk of megaloblastic anemia when taking TMP/SMZ, so withholding this drug in this population is recommended. TMP/SMZ shares hypersensitivity reactions with oral sulfonylurea-type hypoglycemics that are used with type 2 diabetes mellitus, so it is not necessary to assess the blood glucose level more often. TMP/SMZ can cause hyperkalemia, so potassium supplements are contraindicated. Patients taking TMP/SMZ should consume more fluids to maintain renal blood flow and prevent renal damage.

A patient with chronic gout is admitted to the hospital for treatment for an infection. The patient is receiving allopurinol and ampicillin. The nurse is preparing to administer medications and notes that the patient has a temperature of 101°F and a rash. What will the nurse do? a. Withhold the allopurinol and notify the prescriber of the drug reaction. b. Withhold the ampicillin and contact the provider to request a different antibiotic. c. Request an order for an antihistamine to minimize the drug side effects. d. Suggest giving a lower dose of the allopurinol while giving ampicillin.

ANS: A Allopurinol can cause a hypersensitivity syndrome, which is characterized by a rash and fever. If these occur, the drug should be discontinued immediately. The combination of ampicillin and allopurinol increases the risk of this reaction; if it occurs, the allopurinol, not the ampicillin, should be discontinued. Antihistamines are not indicated. Lowering the dose of allopurinol is not indicated.

A prescriber has ordered cefoxitin for a patient who has an infection caused by a gram-negative bacteria. The nurse taking the medication history learns that the patient experienced a maculopapular rash when taking amoxicillin [Amoxil] several years earlier. What will the nurse do? a. Administer the cefoxitin and observe for any side effects. b. Give the cefoxitin and have epinephrine and respiratory support available. c. Request an order for a different, nonpenicillin, noncephalosporin antibiotic. d. Request an order to administer a skin test before giving the cefoxitin.

ANS: A Because of structural similarities between penicillins (PCNs) and cephalosporins, a few patients allergic to one drug type will be allergic to the other drug type, although this is rare. For patients with mild PCN allergy, such as rash, cephalosporins can be used with minimal concern, so it is correct to administer the drug and monitor for side effects. It is unnecessary to prepare for anaphylaxis, to give another class of drug, or to administer a skin test.

A patient is admitted for treatment of gout that has been refractory to treatment with allopurinol and probenecid. The patient is taking colchicine, and the prescriber orders pegloticase [Krystexxa]. Before administering this drug, the nurse will expect to: a. administer an antihistamine and a glucocorticoid. b. discontinue the colchicine. c. increase the dose of colchicine. d. prepare to administer a bronchodilator if needed.

ANS: A Because pegloticase poses a risk of triggering anaphylaxis, patients should be pretreated with an antihistamine and a glucocorticoid. Colchicine is indicated at the initiation of treatment with pegloticase to reduce the intensity of gout flare-ups. Bronchodilators are not indicated.

A child with an ear infection is not responding to treatment with amoxicillin [Amoxil]. The nurse will expect the provider to order: a. amoxicillin-clavulanic acid [Augmentin]. b. ampicillin. c. nafcillin. d. penicillin G [Benzylpenicillin].

ANS: A Beta-lactamase inhibitors are drugs that inhibit bacterial beta-lactamases. These drugs are always given in combination with a penicillinase-sensitive penicillin. Augmentin contains amoxicillin and clavulanic acid and is often used when patients fail to respond to amoxicillin alone. Ampicillin is similar to amoxicillin, but amoxicillin is preferred and, if drug resistance occurs, ampicillin is equally ineffective. Pharmaceutical chemists have developed a group of penicillins that are resistant to inactivation by beta-lactamases (eg, nafcillin), but these drugs are indicated only for penicillinase-producing strains of staphylococci. Penicillin G would be as ineffective as amoxicillin if beta-lactamase is present.

A patient who is receiving intravenous ciprofloxacin for pneumonia develops diarrhea. A stool culture is positive for Clostridium difficile. The nurse will expect the provider to: a. add metronidazole [Flagyl]. b. increase the dose of ciprofloxacin. c. restrict dairy products. d. switch to gemifloxacin.

ANS: A C. difficile is resistant to fluoroquinolones; metronidazole is the drug of choice to treat this infection. Metronidazole is lethal only to anaerobic organisms, so the ciprofloxacin should be continued to treat the pneumonia. Increasing the dose of ciprofloxacin is not indicated, because C. difficile is resistant to ciprofloxacin. Gemifloxacin is approved for use in respiratory infections.

Which side effect of clindamycin [Cleocin] causes the most concern and may warrant discontinuation of the drug? a. Diarrhea b. Headache c. Nausea d. Vomiting

ANS: A CDAD is a serious, sometimes fatal suprainfection associated with clindamycin. Patients with diarrhea should notify their prescriber immediately and discontinue the drug until this condition has been ruled out. Headache, nausea, and vomiting do not warrant discontinuation of the drug and are not associated with severe side effects.

A nurse is explaining to nursing students why a cephalosporin is used in conjunction with an aminoglycoside for a patient with an infection. Which statement by a student indicates understanding of the teaching? a. "Cephalosporins enhance the actions of aminoglycosides by weakening bacterial cell walls." b. "Cephalosporins prevent neuromuscular blockade associated with aminoglycosides." c. "Cephalosporins prolong the postantibiotic effects of the aminoglycosides so doses can be decreased." d. "Cephalosporins reduce bacterial resistance to aminoglycosides."

ANS: A Cephalosporins, penicillins, and vancomycin can be used in conjunction with aminoglycosides; these drugs weaken the bacterial cell wall and enhance the bactericidal actions of aminoglycosides. Cephalosporins do not prevent neuromuscular blockade. They do not prolong the postantibiotic effects of aminoglycosides. They do not affect bacterial resistance.

A nurse is providing teaching for a patient who will begin taking clarithromycin ER [Biaxin XL] to treat an Helicobacter pylori infection. Which statement by the patient indicates understanding of the teaching? a. "I may experience distorted taste when taking this medication." b. "I should take 1 tablet twice daily for 10 days." c. "I should take this medication on an empty stomach." d. "This medication does not interact with other drugs."

ANS: A Clarithromycin is available in an extended-relief preparation as Biaxin XL. Biaxin can cause distortion of taste, so patients should be warned of this side effect. Biaxin XL should be taken once and not twice daily. Biaxin should be taken with food. Biaxin interacts with other drugs by inhibiting hepatic metabolism of those drugs.

A nurse is preparing to administer an antibiotic to a patient with methicillin-resistant Staphylococcus aureus (MRSA). The nurse would expect the healthcare provider to order which antibiotic? a. Daptomycin [Cubicin] b. Levofloxacin [Levaquin] c. Norfloxacin [Noroxin] d. Ciprofloxacin [Cipro]

ANS: A Daptomycin is active against MRSA. Levofloxacin and norfloxacin are not approved to treat MRSA. Ciprofloxacin is a poor choice for staphylococcal infections, including MRSA.

A patient taking risedronate IR [Actonel] for osteoporosis reports experiencing diarrhea and headaches. What will the nurse tell this patient? a. These are common side effects of this drug. b. These symptoms indicate serious toxicity. c. The patient should discuss taking risedronate DR [Atelvia] with the provider. d. The medication should be taken after a meal to reduce symptoms. v

ANS: A Diarrhea and headaches are common adverse effects of risedronate IR. These symptoms do not indicate toxicity. The side effects of Atelvia are similar to those of Actonel. Taking the medication after a meal will not reduce these effects.

A child with an upper respiratory infection caused by B. pertussis is receiving erythromycin ethylsuccinate. After 2 days of treatment, the parent asks the nurse why the child's symptoms have not improved. Which response by the nurse is correct? a. "Erythromycin eliminates the bacteria that causes the infection, but not the toxin that causes the symptoms." b. "We may need to add penicillin or another antibiotic to increase the antimicrobial spectrum." c. "We will need to review the culture sensitivity information to see whether a different antibiotic is indicated." d. "Your child may have developed a suprainfection that we need to culture and treat."

ANS: A Erythromycin is the drug of first choice for treating pertussis infections. Because symptoms are caused by a bacterial toxin and not by the bacteria itself, the drug eliminates the bacteria but does little to alter the course of the disease. It is given to lower infectivity. It is not necessary to add another antibiotic, review the sensitivity information, or look for a suprainfection.

A nurse is providing education to a patient who will begin taking alendronate [Fosamax]. Which complication should the patient be instructed to report immediately? a. Difficulty swallowing b. Dizziness c. Drowsiness d. Pallor

ANS: A Esophagitis is the most serious adverse effect of alendronate, sometimes resulting in ulceration. The nurse should instruct the patient to report difficulty swallowing immediately, because it can be a sign of esophageal injury. Dizziness is not an adverse effect of alendronate. Drowsiness is not a symptom associated with alendronate. Pallor is not a symptom associated with alendronate.

A patient is to undergo orthopedic surgery, and the prescriber will order a cephalosporin to be given preoperatively as prophylaxis against infection. The nurse expects the provider to order which cephalosporin? a. First-generation cephalosporin b. Second-generation cephalosporin c. Third-generation cephalosporin d. Fourth-generation cephalosporin

ANS: A First-generation cephalosporins are widely used for prophylaxis against infection in surgical patients, because they are as effective, less expensive, and have a narrower antimicrobial spectrum than second-, third-, and fourth-generation cephalosporins.

A patient with HIV and mucocutaneous HSV is being treated with foscarnet after failing treatment with acyclovir. After 2 weeks, the patient's dose is increased to 90 mg/kg over 2 hours from 40 mg/kg over 1 hour. The patient reports numbness in the extremities and perioral tingling. What will the nurse do? a. Notify the provider and request an order for a serum calcium level. b. Notify the provider of potential foscarnet overdose. c. Request an order for a creatinine clearance level. d. Request an order of IV saline to be given before the next dose.

ANS: A Foscarnet frequently causes hypocalcemia and other electrolyte and mineral imbalances. Paresthesias, numbness in the extremities, and perioral tingling can indicate hypocalcemia, so a calcium level should be drawn. These are not signs of foscarnet overdose. Nephrotoxicity may occur, but these are not signs of renal complications, so a creatinine clearance is not indicated. If nephrotoxicity occurs, prehydration with IV saline is indicated to reduce the risk of renal injury.

A patient who is taking gentamicin and a cephalosporin for a postoperative infection requests medication for mild postsurgical pain. The nurse will expect to administer which of the following medications? a. Acetaminophen b. Aspirin c. Ibuprofen d. Morphine

ANS: A Gentamicin and cephalosporins are both nephrotoxic. This patient should avoid taking other potentially nephrotoxic drugs. Acetaminophen is not nephrotoxic and may be given for mild pain. Aspirin and ibuprofen are both nephrotoxic. Morphine is not nephrotoxic but is not indicated for mild pain.

A nurse transcribes a new prescription for potassium penicillin G given intravenously (IV) every 8 hours and gentamicin given IV every 12 hours. Which is the best schedule for administering these drugs? a. Give the penicillin at 0800, 1600, and 2400; give the gentamicin [Garamycin] at 1800 and 0600. b. Give the penicillin at 0800, 1600, and 2400; give the gentamicin [Garamycin] at 1200 and 2400. c. Give the penicillin at 0600, 1400, and 2200; give the gentamicin [Garamycin] at 0600 and 1800. d. Give the penicillin every 8 hours; give the gentamicin [Garamycin] simultaneously with two of the penicillin doses.

ANS: A Gentamicin should never be administered concurrently with penicillin, because they will interact, and the penicillin may inactivate the aminoglycoside. All the other options show concurrent administration.

A patient is diagnosed with an infection caused by Staphylococcus aureus, and the prescriber orders intravenous gentamicin and penicillin (PCN). Both drugs will be given twice daily. What will the nurse do? a. Administer gentamicin, flush the line, and then give the penicillin. b. Give the gentamicin intravenously and the penicillin intramuscularly. c. Infuse the gentamicin and the penicillin together to prevent fluid overload. d. Request an order to change the penicillin to vancomycin.

ANS: A Gentamicin should not be infused with penicillins in the same solution, because PCN inactivates gentamicin; therefore, the nurse should give one first, flush the line, and then give the other. The nurse cannot give a drug IM when it is ordered IV without an order from the prescriber. These two drugs should not be infused in the same solution. There is no indication for changing the PCN to vancomycin; that should be done for serious infections.

A patient will begin taking hydroxychloroquine [Plaquenil] for rheumatoid arthritis. The patient is currently taking high-dose NSAIDs and methotrexate. What will the nurse teach the patient? a. That an eye exam is necessary at the beginning of therapy with this drug b. That the dose of NSAIDs may be decreased when beginning hydroxychloroquine c. To obtain tests of renal and hepatic function while taking this drug d. To stop taking methotrexate when starting hydroxychloroquine

ANS: A Hydroxychloroquine can cause retinal damage so an eye exam is necessary at the onset of treatment as well as every 6 months during treatment. Patients taking other drugs should continue to take those when beginning treatment with hydroxychloroquine since full therapeutic effects take months to develop. Renal and hepatic toxicity are not concerns. The drug is usually combined with methotrexate.

A nurse is discussing methicillin-resistant Staphylococcus aureus (MRSA) with a group of nursing students. Which statement by a student correctly identifies the basis for MRSA resistance? a. "MRSA bacteria have developed PBPs with a low affinity for penicillins." b. "MRSA bacteria produce penicillinases that render penicillin ineffective." c. "MRSA occurs because of host resistance to penicillins." d. "MRSA strains replicate faster than other Staphylococcus aureus strains."

ANS: A MRSA strains have a unique mechanism of resistance, which is the production of PBPs with a low affinity for penicillins and all other beta-lactam antibiotics. MRSA resistance is not related to beta-lactamase production. MRSA resistance refers to bacterial and not host resistance. The resistance of MRSA strains is not related to speed of replication.

A nurse is preparing to administer IV calcium chloride to a patient with a low serum calcium level. Which drug on the patient's medication record, administered concurrently, would require additional patient monitoring by the nurse? a. Digoxin [Lanoxin] b. Furosemide [Lasix] c. Lorazepam [Ativan] d. Pantoprazole [Protonix]

ANS: A Parenteral calcium may cause severe bradycardia in patients taking digoxin; therefore, the heart rate should be monitored closely. Concurrent administration of calcium chloride and pantoprazole, lorazepam, or furosemide is not known to lead to drug interactions.

A patient is beginning therapy with oral methotrexate [Rheumatrex] for rheumatoid arthritis. The nurse will teach this patient about the importance of: a. having routine renal and hepatic function tests. b. limiting folic acid consumption. c. reporting alopecia and rash. d. taking the medication on a daily basis.

ANS: A Periodic tests of renal and liver function are mandatory for patients taking methotrexate. Patients taking methotrexate should take folic acid supplements. Alopecia and rash are not worrisome side effects. Methotrexate is taken once weekly.

A patient has lamivudine-resistant hepatitis B and has been taking entecavir [Baraclude] for 2 years. The patient asks the nurse why the provider has recommended taking the drug for another year. What will the nurse tell the patient? a. "Entecavir can reverse fibrosis and cirrhosis of the liver when taken long term." b. "It is necessary to continue taking entecavir to avoid withdrawal symptoms." c. "The drug will be given until the infection is completely eradicated." d. "You will need to continue taking entecavir to prevent lactic acidosis and hepatotoxicity." v

ANS: A Recent evidence indicates that, with long-term use (3 years), entecavir can reverse fibrosis and cirrhosis. The drug is not continued to avoid withdrawal symptoms. Patients who stop taking entecavir may experience acute exacerbations of hepatitis B; the disease is not eradicated. Entecavir can cause lactic acidosis and hepatotoxicity; it does not prevent these adverse effects

A patient has a Pseudomonas aeruginosa infection that is sensitive to aminoglycosides, and the prescriber orders gentamicin. The patient tells the nurse that a friend received amikacin [Amikin] for a similar infection and wonders why amikacin was not ordered. What will the nurse tell the patient? a. "Amikacin is given when infectious agents are resistant to other aminoglycosides." b. "Amikacin is more vulnerable to inactivation by bacterial enzymes." c. "Amikacin is a narrow-spectrum drug and will probably not work for this infection." d. "Gentamicin is less toxic to the ears and the kidneys."

ANS: A Resistance to amikacin is uncommon at this point; to minimize the emergence of amikacin-resistant bacteria, this drug is reserved for infections in which resistance to other aminoglycosides has developed. Amikacin is the least susceptible to inactivation by bacterial enzymes. Amikacin is a broad-spectrum antibiotic. All aminoglycosides are ototoxic and nephrotoxic.

A nurse is discussing the administration of an intravenous infusion of rituximab (Rituxan) with a nursing student. Which statement by the student indicates a need for further education about the care of a patient receiving this drug? a. "Angioedema and hypersensitivity may occur, but they are usually self-limiting and mild." b. "I should be prepared to administer epinephrine, glucocorticoids, and oxygen if needed." c. "I will administer an antihistamine and acetaminophen before beginning the infusion." d. "I will monitor this patient's blood pressure, respiratory rate, and oxygen saturation closely."

ANS: A Rituximab can cause severe infusion-related hypersensitivity reactions. Nurses should be prepared to administer epinephrine, steroids, and O2 if needed. Antihistamines and acetaminophen are given before infusion. Close monitoring of vital signs and oxygenation are indicated.

A patient will be discharged from the hospital with a prescription for TMP/SMZ [Bactrim]. When providing teaching for this patient, the nurse will tell the patient that it will be important to: a. drink 8 to 10 glasses of water each day. b. eat foods that are high in potassium. c. take the medication with food. d. take folic acid supplements.

ANS: A TMP/SMZ can injure the kidneys, because it causes deposition of sulfonamide crystals in the kidneys. Patients should be advised to drink 8 to 10 glasses of water a day to maintain a urine flow of 1200 mL in adults. Trimethoprim can cause hyperkalemia, so consuming extra potassium is unnecessary. The medication should be taken on an empty stomach. It is not necessary to consume extra folic acid, because mammalian cells use dietary folate and do not have to synthesize it; it is the process of folic acid synthesis that is altered by sulfonamides.

A pregnant adolescent patient asks the nurse whether she should continue to take her prescription for tetracycline [Sumycin] to clear up her acne. Which response by the nurse is correct? a. "Tetracycline can be harmful to the baby's teeth and should be avoided." b. "Tetracycline is safe to take during pregnancy." c. "Tetracycline may cause allergic reactions in pregnant women." d. "Tetracycline will prevent asymptomatic urinary tract infections."

ANS: A Tetracyclines can cause discoloration of deciduous teeth of infants if taken by the mother after the fourth month of gestation. Tetracyclines should not be given to pregnant women. Tooth discoloration can be prevented if the drugs are not taken by pregnant women or by children under 8 years of age. Tetracycline is not appropriate for a pregnant patient. Pregnancy does not precipitate an allergic response to tetracycline. Tetracycline should not be used to prevent urinary tract infections (UTIs), especially in pregnant women.

A patient with severe glucocorticoid-induced osteoporosis will start therapy with teriparatide [Forteo]. What will the nurse expect to administer? a. 20 mcg once daily subQ b. 20 mcg twice daily subQ c. 10 mcg once daily subQ d. 10 mcg twice daily subQ

ANS: A The dose of teriparatide for all indications is 20 mcg once daily subQ.

A male patient with hepatitis C will begin triple drug therapy with pegylated interferon alfa 2a [Pegasys], ribavirin [Ribasphere], and boceprevir [Victrelis]. The patient tells the nurse that his wife is pregnant. What will the nurse tell him? a. Boceprevir is contraindicated in males whose partners are pregnant. b. He should use a barrier contraceptive when having sex. c. He should use dual drug therapy with pegylated interferon alfa and ribavirin only. d. This combination drug therapy is safe for him to use.

ANS: A The triple combination is dangerous for pregnant women whose partners are using it, so it is contraindicated for any man whose partner is pregnant. Barrier contraceptives should be used by couples to prevent pregnancy when either partner is taking the triple combination therapy. Ribavirin is teratogenic and is not safe when a partner is pregnant. This combination is not safe for pregnant women whose partners are taking these drugs.

A patient with no known drug allergies is receiving amoxicillin [Amoxil] PO twice daily. Twenty minutes after being given a dose, the patient complains of shortness of breath. The patient's blood pressure is 100/58 mm Hg. What will the nurse do? a. Contact the provider and prepare to administer epinephrine. b. Notify the provider if the patient develops a rash. c. Request an order for a skin test to evaluate possible PCN allergy. d. Withhold the next dose until symptoms subside.

ANS: A This patient is showing signs of an immediate penicillin allergy, that is, one that occurs within 2 to 30 minutes after administration of the drug. The patient is showing signs of anaphylaxis, which include laryngeal edema, bronchoconstriction, and hypotension; these must be treated with epinephrine. This is an emergency, and the provider must be notified immediately, not when other symptoms develop. It is not necessary to order skin testing. The patient must be treated immediately, and subsequent doses should not be given.

A patient will be discharged home to complete treatment with intravenous cefotetan with the assistance of a home nurse. The home care nurse will include which instruction when teaching the patient about this drug treatment? a. Abstain from alcohol consumption during therapy. b. Avoid dairy products while taking this drug. c. Take an antihistamine if a rash occurs. d. Use nonsteroidal anti-inflammatory drugs (NSAIDs), not acetaminophen, for pain.

ANS: A Two cephalosporins, including cefotetan, can induce a state of alcohol intolerance and cause a disulfiram-like reaction when alcohol is consumed; therefore, patients should be advised to avoid alcohol. It is not necessary to avoid dairy products. Patients who experience a rash should report this to their provider. Cefotetan can also promote bleeding, so drugs that inhibit platelet aggregation should be avoided.

A provider has ordered ceftriaxone 4 gm once daily for a patient with renal impairment. What will the nurse do? a. Administer the medication as prescribed. b. Contact the provider to ask about giving the drug in divided doses. c. Discuss increasing the interval between doses with the provider. d. Discuss reducing the dose with the provider.

ANS: A Unlike other cephalosporins, ceftriaxone is eliminated largely by the liver, so dosage reduction is unnecessary in patients with renal impairment. Giving the drug in divided doses, increasing the interval between doses, and reducing the dose are not necessary.

A nurse is discussing the role of vitamin D in calcium regulation with a nursing student. Which statement by the student indicates a need for further teaching? a. "Adequate amounts of vitamin D occur naturally in the diet." b. "Vitamin D3 is preferred over vitamin D2." c. "Vitamin D can promote bone decalcification." d. "Vitamin D increases the absorption of calcium and phosphorus from the intestine."

ANS: A Vitamin D does not occur naturally in the diet. Adequate amounts are gained through fortified foods, supplements, and exposure to sunlight. Vitamin D3 is preferred. If calcium intake is not sufficient, vitamin D can promote bone decalcification. Vitamin D acts to increase the absorption of calcium and phosphorus from the intestine.

A patient is receiving gentamicin once daily. A nursing student asks the nurse how the drug can be effective if given only once a day. The nurse explains drug dosing schedules for aminoglycosides. Which statement by the student indicates a need for further teaching? a. "Gentamicin has a longer half-life than other aminoglycosides." b. "Large doses given once daily yield higher peak levels." c. "The postantibiotic effect lasts for several hours." d. "There is less risk of ototoxicity and nephrotoxicity with large daily doses."

ANS: A When a daily dose is given once daily instead of divided into 2 or 3 doses, a higher peak level can be achieved. The higher peak, along with the fact that aminoglycosides have a postantibiotic effect, means that the bacterial kill is just as great with one dose as with 2 or 3 doses per day. When a single daily dose is given, the risk of toxicity is reduced. Gentamicin does not have a longer half-life than other aminoglycosides.

A patient is admitted to the unit for treatment for an infection. The patient receives IV amikacin [Amikin] twice a day. When planning for obtaining a peak aminoglycoside level, when should the nurse see that the blood is drawn? a. 30 minutes after the IV infusion is complete b. 1 hour after the IV infusion is complete c. 1 hour before administration of the IV infusion d. A peak level is not indicated with twice-daily dosing.

ANS: A When divided daily doses are used, blood samples for measurement of peak levels are drawn 1 hour after IM injection and 30 minutes after completion of an IV infusion. This medication is administered IV, so blood draws must follow 30 minutes after infusion to obtain peak levels. Measurement of peak levels is unnecessary only when a single daily dose is used.

A patient has an infection caused by Pseudomonas aeruginosa. The prescriber has ordered piperacillin and amikacin, both to be given intravenously. What will the nurse do? a. Make sure to administer the drugs at different times using different IV tubing. b. Suggest giving larger doses of piperacillin and discontinuing the amikacin. c. Suggest that a fixed-dose combination of piperacillin and tazobactam [Zosyn] be used. d. Watch the patient closely for allergic reactions, because this risk is increased with this combination.

ANS: A When penicillins are present in high concentrations, they interact with aminoglycosides and inactivate the aminoglycoside; therefore, these two drugs should never be mixed in the same IV solution. The drugs should be given at different times with different tubing. In the treatment of Pseudomonas infections, extended-spectrum penicillins, such as piperacillin, usually are given in conjunction with an antipseudomonal aminoglycoside, such as amikacin; therefore, suggesting a larger dose of piperacillin and discontinuation of the amikacin is incorrect. Zosyn is not recommended. The risk of allergic reactions does not increase with this combination of drugs.

A patient is diagnosed with a lung infection caused by P. aeruginosa. The culture and sensitivity report shows sensitivity to all aminoglycosides. The nurse knows that the rate of resistance to gentamicin is common in this hospital. The nurse will expect the provider to order which medication? a. Amikacin [Amikin] b. Gentamicin c. Paromomycin d. Tobramycin

ANS: A When resistance to gentamicin and tobramycin is common, amikacin is the drug of choice for initial treatment of aminoglycoside-sensitive infections. Gentamicin would not be indicated, because resistance is more likely to develop. Paromomycin is used only for local effects within the intestine and is given orally. Tobramycin is not indicated, because organisms can more readily develop resistance.

A nursing student wants to know the differences between hospital-associated methicillin-resistant Staphylococcus aureus (HA-MRSA) and community-associated methicillin-resistant Staphylococcus aureus (CA-MRSA). Which statements about CA-MRSA are true? (Select all that apply.) a. 20% to 30% of the general population are colonized with CA-MRSA. b. Boils caused by CA-MRSA can be treated without antibiotics. c. CA-MRSA is less dangerous than HA-MRSA. d. CA-MRSA does not cause necrotizing fasciitis. e. CA-MRSA is transmitted by airborne droplets.

ANS: A, B, C CA-MRSA is thought to be present in 20% to 30% of the population, and many of these individuals are asymptomatic carriers. Boils caused by CA-MRSA can often be treated by surgical drainage alone. CA-MRSA is less dangerous than HA-MRSA but more dangerous than methicillin-sensitive Staphylococcus aureus (MSSA). CA-MRSA generally causes mild skin infections but can cause more serious infections, such as necrotizing fasciitis. CA-MRSA is transmitted by skin-to-skin contact and by contact with contaminated objects.

Which methods have been identified for preventing antimicrobial resistance? (Select all that apply.) a. Administering influenza vaccine b. Stopping antibiotic therapy as soon as possible when treating infection c. Treating bacterial colonization in asymptomatic patients d. Using narrow-spectrum antibiotics when possible e. Using national data to determine antimicrobial drug selection

ANS: A, B, D Vaccines help prevent disease and thus reduce the need to use antimicrobial drugs. Discontinuing antibiotic therapy as soon as possible, such as when an infection has been fully treated or when an infection is unlikely, helps reduce exposure of normal flora and other microbes to antimicrobials. Narrow-spectrum antibiotics target the pathogen and are less likely to act against other microbes. It is not correct to treat bacterial colonization when patients are asymptomatic. Local data give clearer information on local trends in susceptibility and resistance and thus are more likely to guide antimicrobial drug selection correctly.

Which patients may receive gentamicin safely? (Select all that apply.) a. A 6-day-old newborn b. A 5-year-old child c. A pregnant woman d. A woman breast-feeding an infant e. An elderly patient with renal disease

ANS: A, B, D Aminoglycosides may be safely given to infants younger than 8 days old and to children and adolescents. Use during pregnancy is not recommended because they may harm the fetus. Gentamicin is probably safe during lactation. Gentamicin is used with caution in older patients with reduced renal function.

Besides the cost of administering a given drug, which are considerations when a provider selects a cephalosporin to treat an infection? (Select all that apply.) a. Adverse effects b. Antimicrobial spectrum c. Brand name d. Manufacturer e. Pharmacokinetics

ANS: A, B, E Cephalosporins tend to be more alike than different, but some differences make some preferable to others. Adverse effects, antimicrobial spectrum, and drug pharmacokinetics affect a prescriber's decision to select one drug over another. The agent's brand name and manufacturer should not have anything to do with drug selection.

Which infection(s) may be treated with linezolid [Zyvox])? (Select all that apply.) a. Community-acquired pneumonia (CAP) that is penicillin sensitive b. Nosocomial pneumonia caused by methicillin-sensitive Staphylococcus aureus (MSSA) c. Pneumonias caused by Mycoplasma avium d. Superficial methicillin-resistant Staphylococcus aureus skin infections (MRSA) e. Vancomycin-resistant infections

ANS: A, B, E Linezolid is indicated for CAP caused by PCN-sensitive strains of Streptococcus pneumoniae, nosocomial pneumonia caused by MSSA and MRSA, and vancomycin-resistant enterococcal (VRE) infections. It is not recommended for M. avium infections or for superficial skin infections caused by MRSA.

The nurse is teaching a pharmacology refresher course to a group of nurses. A student asks what host factors affect the choice of agents in antimicrobial therapy. The nurse will tell the students that such host factors include what? (Select all that apply.) a. Age b. Gender c. Immune system status d. Infection site e. Weight

ANS: A, C, D Host factors, such as the patient's age, immune status, and the site of the infection, affect the choice of antimicrobial therapy used to treat an infection. Gender and weight do not affect the choice of antimicrobial but may affect the dose and route of administration.

Tetracyclines are considered first-line drugs for which disorder(s)? (Select all that apply.) a. Chlamydia trachomatis cervicitis b. Clostridium difficile diarrhea c. Lyme disease d. Methicillin-resistant Staphylococus aureus skin infections e. Typhus fever

ANS: A, C, E Tetracyclines are drugs of first choice for rickettsial diseases, infections caused by C. trachomatis, brucellosis, cholera, Mycoplasma pneumonia, Lyme disease, anthrax, and gastric infections caused by H. pylori. They are not first-line drugs for CDAD or MRSA skin infections.

Which antibiotics may be administered topically? (Select all that apply.) a. Bacitracin b. Daptomycin c. Ofloxacin d. Polymyxin B e. Rifampin

ANS: A, D Bacitracin and Polymyxin B are both topical antibiotics. Daptomycin, ofloxacin, and rifampin are not formulated to be administered topically.

Which drugs are approved for treating osteoporosis in men? (Select all that apply.) a. Alendronate [Fosamax] b. Calcitonin c. Raloxifene [Evista] d. Teriparatide [Forteo] e. Zoledronate [Reclast]

ANS: A, D, E Only five drugs have been approved to treat osteoporosis in men, including alendronate, teriparatide, and zoledronate. Calcitonin has been tried, but without proof of efficacy. Raloxifene is a SERM, used in women only.

A child has juvenile idiopathic arthritis and will begin a course of etanercept (Enbrel). The nurse will conduct a thorough health history with special attention to: a. bone growth development. b. immunization status. c. pulmonary function. d. seasonal allergies.

ANS: B Patients beginning therapy with etanercept should be fully immunized to prevent the risk of communicable diseases as well as to avoid giving live-virus vaccines, since etanercept suppresses immunity. Bone growth development, pulmonary function, and seasonal allergies are not of special concern with this drug.

A patient is receiving an intraperitoneal aminoglycoside during surgery. To reverse a serious side effect of this drug, the nurse may expect to administer which agent? a. Amphotericin B b. Calcium gluconate c. Neuromuscular blocker d. Vancomycin

ANS: B Aminoglycosides can inhibit neuromuscular transmission, especially during intraperitoneal or intrapleural instillation, and this risk is increased when neuromuscular blocking agents and general anesthetics are given. Calcium can reverse neuromuscular blockade. Amphotericin B, additional neuromuscular blockers, and vancomycin are not indicated.

A patient who will begin taking colchicine for gout reports taking nonsteroidal anti-inflammatory drugs, simvastatin, amoxicillin, and digoxin. What will the nurse do? a. Contact the provider to discuss using a different antibiotic while this patient is taking colchicine. b. Notify the provider about the potential risk of muscle injury when simvastatin is taken with colchicine. c. Request an order for cardiorespiratory monitoring, because the patient is taking digoxin. d. Suggest that the nonsteroidal anti-inflammatory drugs (NSAIDs) be withdrawn during colchicine therapy.

ANS: B Colchicine can cause rhabdomyolysis, and this risk is increased in patients who also take simvastatin or other statin drugs. Amoxicillin does not interact with colchicine. The side effects of digoxin are not increased by concurrent use with colchicine. NSAIDs can safely be taken with colchicine.

A patient who is hospitalized for an acute gout attack has received several doses of hourly oral colchicine but still reports moderate to severe pain. As the nurse prepares to administer the next dose, the patient begins vomiting. What will the nurse do? a. Contact the provider to discuss giving a lower dose of colchicine. b. Hold the medication and notify the prescriber. c. Explain that this is a common side effect that will soon stop. d. Request an order for an antiemetic so that the next dose of colchicine may be given.

ANS: B Colchicine should be discontinued immediately, regardless of the status of the joint pain, if gastrointestinal (GI) symptoms occur. The patient's symptoms indicate injury to the GI endothelium. Once damage begins to occur, lowering the dose is not indicated. GI toxicity will not abate over time. An antiemetic may be useful for stopping the vomiting; however, continued administration of the drug can lead to further damage to the GI endothelium.

1. A patient who has been diagnosed with rheumatoid arthritis (RA) for 1 month and has generalized symptoms is taking high-dose nonsteroidal anti-inflammatory drugs (NSAIDs) and an oral glucocorticoid. The provider has ordered methotrexate [Rheumatrex]. The patient asks the nurse why methotrexate is necessary since pain and swelling have been well controlled with the other medications. The nurse will tell the patient that: a. a methotrexate regimen can reduce overall costs and side effects of treatment. b. starting methotrexate early can help delay joint degeneration. c. starting methotrexate now will help increase life expectancy. d. with methotrexate, doses of NSAIDs can be reduced to less toxic levels.

ANS: B Current guidelines for treatment of RA recommend starting a disease-modifying antirheumatic drug (DMARD) early—within 3 months of diagnosis for most patients—in order to delay joint degeneration. Methotrexate may take up to 3 to 6 weeks to be at therapeutic levels, so NSAIDs and glucocorticoids should be continued until this occurs. Methotrexate is expensive and has more toxic side effects. Patients taking methotrexate have been shown in some data to have decreased life expectancy. Patients may eventually be able to stop taking NSAIDs altogether.

A patient with hepatitis B begins treatment with adefovir [Hepsera] and asks the nurse how long the drug therapy will last. The nurse will tell the patient that the medication will need to be taken for: a. a lifetime. b. an indefinite, prolonged period of time. c. 48 weeks. d. until nephrotoxicity occurs.

ANS: B Current guidelines recommend treatment only for patients at highest risk; it is unknown whether treatment should continue lifelong. Treatment is usually prolonged, without a specific period of time. Nephrotoxicity is common but is not the deciding factor when determining length of effective treatment.

A patient has a positive test for hepatitis C and is admitted to the hospital. The admission laboratory tests reveal a normal ALT, and a liver biopsy is negative for hepatic fibrosis and inflammation. The nurse will prepare this patient for: a. dual therapy with pegylated interferon alfa and ribavirin. b. no medication therapy at this time. c. pegylated interferon alfa only until ALT levels are elevated. d. triple drug therapy with pegylated interferon alfa, ribavirin, and boceprevir.

ANS: B Current recommendations are that treatment is used only for patients with HCV viremia, persistent elevation of ALT, and evidence of hepatic fibrosis and inflammation upon liver biopsy. Dual therapy has been the regimen of choice for patients with the above symptoms, but the addition of a protease inhibitor has been shown to improve outcomes. It is not correct to give pegylated interferon alfa until ALT levels are elevated. Triple drug therapy is used for patients with the above symptoms.

A patient will receive oral ciprofloxacin [Cipro] to treat a urinary tract infection. The nurse provides teaching for this patient. Which statement by the patient indicates a need for further teaching? a. "I may have abdominal pain and nausea, but these are usually mild." b. "I should take this medication with food or milk to improve absorption." c. "I should stop taking the medication immediately if I experience heel pain." d. "I will need to use sunscreen every time I go outdoors."

ANS: B Dairy products inhibit the absorption of ciprofloxacin, so they should be avoided. Abdominal pain and nausea and vomiting are common and usually mild. Patients should stop taking the drug if heel pain occurs until tendonitis has been ruled out. Photosensitivity can occur, so sunscreen should be used.

A patient is taking erythromycin ethylsuccinate for a chlamydial infection and develops vaginal candidiasis. The prescriber orders ketoconazole to treat the superinfection. What will the nurse do? a. Administer the erythromycin and the ketoconazole as ordered. b. Contact the provider to discuss changing to a different antifungal medication. c. Contact the provider to discuss increasing the dose of erythromycin. d. Contact the provider to suggest using erythromycin stearate.

ANS: B Erythromycin can prolong the QT interval when present in large concentrations. When erythromycin is combined with a CYP3A4 inhibitor, such as ketoconazole, the risk of sudden cardiac death increases fivefold. The nurse should discuss changing the antifungal medication to one that is not a CYP3A4 inhibitor. It is not correct to give the ketoconazole without questioning the order. Increasing the dose of erythromycin would increase the risk of QT prolongation. Changing to a different preparation of erythromycin would not alter the risk.

A patient who takes the loop diuretic ethacrynic acid is given intravenous gentamicin for an infection. After several days of treatment with gentamicin, the nurse reviews the patient's most recent laboratory results and notes a gentamicin trough of 2.1 mcg/mL and normal blood urea nitrogen (BUN) and serum creatinine levels. The nurse will question the patient about: a. gastrointestinal (GI) symptoms. b. headache, dizziness, or vertigo. c. presence of rash. d. urine output.

ANS: B Ethacrynic acid has ototoxic properties, and patients who take this drug with an aminoglycoside have an increased risk of ototoxicity, especially when trough levels of the aminoglycoside are elevated. A trough level of 2.1 mcg/mL is above normal limits for gentamicin, so this patient should be asked about early signs of ototoxicity. There is no indication to evaluate for GI symptoms, rash, or urine output.

A patient with HIV contracts herpes simplex virus (HSV), and the prescriber orders acyclovir [Zovirax] 400 mg PO twice daily for 10 days. After 7 days of therapy, the patient reports having an increased number of lesions. The nurse will expect the provider to: a. extend this patient's drug therapy to twice daily for 12 months. b. give intravenous foscarnet every 8 hours for 2 to 3 weeks. c. increase the acyclovir dose to 800 mg PO 5 times daily. d. order intravenous valacyclovir [Valtrex] 1 gm PO twice daily for 10 days.

ANS: B Foscarnet is active against all known herpesviruses and is used in immunocompromised patients with acyclovir-resistant HSV or VZV. This patient is demonstrating resistance to acyclovir, so extending acyclovir therapy or increasing the acyclovir dose will not be effective. Valacyclovir is not approved for use in immunocompromised patients because of the risk for thrombotic thrombocytopenic purpura/hemolytic uremic syndrome.

A woman complains of burning on urination and increased frequency. The patient has a history of frequent urinary tract infections (UTIs) and is going out of town in 2 days. To treat the infection quickly, the nurse would expect the healthcare provider to order: a. aztreonam [Azactam]. b. fosfomycin [Monurol]. c. trimethoprim/sulfamethoxazole [Bactrim]. d. vancomycin [Vancocin].

ANS: B Fosfomycin has been approved for single-dose therapy of UTIs in women. Vancomycin and aztreonam are not indicated for UTIs. Bactrim is indicated for UTIs, but administration of a single dose is not therapeutic.

A patient who has been taking gentamicin for 5 days reports a headache and dizziness. What will the nurse do? a. Request an order for a gentamicin peak level. b. Suspect ototoxicity and notify the prescriber. c. Tell the patient to ask for help with ambulation. d. Tell the patient to report any tinnitus.

ANS: B Headache and dizziness are signs of ototoxicity, and the prescriber should be notified. A peak level is not indicated; it is more important to know the trough level. Telling the patient to ask for help with ambulation and to report tinnitus should both be done but neither one is the priority nursing action.

A nurse is teaching a group of nursing students about influenza prevention. Which statement by a student indicates understanding of the teaching? a. "I may develop a mild case of influenza if I receive the vaccine by injection." b. "I should receive the vaccine every year in October or November." c. "If I have a cold I should postpone getting the vaccine." d. "The antiviral medications are as effective as the flu vaccine for preventing the flu."

ANS: B Influenza vaccine should be given every year in October or November. The vaccine will not cause influenza. Minor illnesses, such as a cold, are not a contraindication for receiving the vaccine. Antiviral medications are not as effective as the flu vaccine in preventing influenza.

A patient has a positive test for influenza type A and tells the nurse that symptoms began 5 days before being tested. The prescriber has ordered oseltamivir [Tamiflu]. The nurse will tell the patient that oseltamivir: a. may decrease symptom duration by 2 or 3 days. b. may not be effective because of the delay in starting treatment. c. may reduce the severity but not the duration of symptoms. d. will alleviate symptoms within 24 hours of the start of therapy.

ANS: B Oseltamivir is most effective when begun within 2 days after symptom onset. When started within 12 hours of symptom onset, it may decrease duration of symptoms by 2 to 3 days. The drug reduces both symptom severity and symptom duration when used in a timely fashion. It does not rapidly alleviate symptoms.

A nurse assisting a nursing student with medications asks the student to describe how penicillins (PCNs) work to treat bacterial infections. The student is correct in responding that penicillins: a. disinhibit transpeptidases. b. disrupt bacterial cell wall synthesis. c. inhibit autolysins. d. inhibit host cell wall function.

ANS: B PCNs weaken the cell wall, causing bacteria to take up excessive amounts of water and subsequently rupture. PCNs inhibit transpeptidases and disinhibit autolysins. PCNs do not affect the cell walls of the host.

A nurse is teaching a nursing student about dalfopristin/quinupristin [Synercid]. Which statement by the student indicates an understanding of the teaching? a. "Patients should stop taking the drug if they experience joint and muscle pain." b. "Patients taking this drug should have blood tests performed frequently." c. "Patients who are allergic to penicillin should not take this drug." d. "This drug will be administered intravenously over a 30- to 60-minute period."

ANS: B Patients taking dalfopristin/quinupristin should have blood levels measured twice the first week and then weekly thereafter to assess for hepatotoxicity. Joint and muscle pain are not an indication for withdrawing the drug. There is no cross-sensitivity to penicillin. The drug is given intravenously over a period of at least 1 hour.

A patient receiving a cephalosporin develops a secondary intestinal infection caused by Clostridium difficile. What is an appropriate treatment for this patient? a. Adding an antibiotic, such as vancomycin [Vancocin], to the patient's regimen b. Discontinuing the cephalosporin and beginning metronidazole [Flagyl] c. Discontinuing all antibiotics and providing fluid replacement d. Increasing the dose of the cephalosporin and providing isolation measures

ANS: B Patients who develop C. difficile infection (CDI) as a result of taking cephalosporins or other antibiotics need to stop taking the antibiotic in question and begin taking either metronidazole or vancomycin. Adding one of these antibiotics without withdrawing the cephalosporin is not indicated. CDI must be treated with an appropriate antibiotic, so stopping all antibiotics is incorrect. Increasing the cephalosporin dose would only aggravate the CDI.

A nurse is preparing to administer a dose of gentamicin to a patient who is receiving the drug 3 times daily. The nurse will monitor ____ levels. a. peak b. peak and trough c. serum drug d. trough

ANS: B When divided doses of aminoglycosides are given, it is important to measure both peak and trough levels of the drug, because it is more difficult to achieve therapeutic peaks in lower doses without causing toxicity. Trough levels are drawn when single-dosing regimens are used, because high peak levels are guaranteed.

The nurse is caring for a patient who is receiving vancomycin [Vancocin]. The nurse notes that the patient is experiencing flushing, rash, pruritus, and urticaria. The patient's heart rate is 120 beats per minute, and the blood pressure is 92/57 mm Hg. The nurse understands that these findings are consistent with: a. allergic reaction. b. red man syndrome. c. rhabdomyolysis. d. Stevens-Johnson syndrome.

ANS: B Rapid infusion of vancomycin can cause flushing, rash, pruritus, urticaria, tachycardia, and hypotension, a collection of symptoms known as red man syndrome. Rhabdomyolysis is not associated with the administration of vancomycin. The patient's symptoms may seem to indicate an allergic reaction, but this is specifically red man syndrome. The symptoms are not those of Stevens-Johnson syndrome, which manifests as blisters or sores (or both) on the lips and mucous membranes after exposure to the sun.

A patient with severe community-acquired pneumonia has been prescribed telithromycin [Ketek]. Which aspect of the patient's medical history is of concern to the nurse? a. Anemia b. Myasthenia gravis c. Renal disease d. Strep. pneumoniae infection

ANS: B Telithromycin is a macrolide antibiotic used only for CAP. Patients with myasthenia gravis may experience rapid muscle weakness after taking the drug, and some have died from respiratory failure, so patients with MG should not take this drug. This drug does not have significant myelosuppression, so anemia is not a concern. The drug causes liver injury, so liver disease, and not renal disease, is a concern. Telithromycin is indicated for treatment of S. pneumonia.

A patient who takes teriparatide [Forteo] administers it subcutaneously with a prefilled pen injector. The patient asks why she must use a new pen every 28 days when there are doses left in the syringe. Which is the correct response by the nurse? a. "Go ahead and use the remaining drug; I know it is so expensive." b. "The drug may not be stable after 28 days." c. "You are probably not giving the drug accurately." d. "You should be giving the drug more frequently.

ANS: B Teriparatide is supplied in 3-mL injectors. The pen should be stored in the refrigerator and discarded after 28 days, even if some drug remains in the syringe. Although the drug is expensive, it is not correct to use what is in the syringe after 28 days. Drug may be left in the syringe even with correct dosing.

A nurse provides teaching for a woman who will begin taking supplemental calcium. Which statement by the woman indicates understanding of the teaching? a. "Chewable calcium tablets are not absorbed well and are not recommended." b. "I should not take more than 600 mg of calcium at one time." c. "I should take enough supplemental calcium to provide my total daily requirements." d. "If I take calcium with green, leafy vegetables, it will increase absorption."

ANS: B To help ensure adequate absorption of calcium, no more than 600 mg should be consumed at one time. Chewable calcium tablets are recommended because of their more consistent bioavailability. The amount of supplemental calcium should be enough to compensate for what is not consumed in the diet and should not constitute the total amount needed per day. Green, leafy vegetables reduce the absorption of calcium.

A patient is diagnosed with periodontal disease, and the provider orders oral doxycycline [Periostat]. The patient asks the purpose of the drug. What is the nurse's response? a. "It is used because of its anti-inflammatory effects." b. "It inhibits collagenase to protect connective tissue in the gums." c. "It reduces bleeding and the pocket depth of oral lesions." d. "It suppresses bacterial growth in the oral mucosa."

ANS: B Two tetracyclines are used for periodontal disease. Doxycycline inhibits collagenase, which destroys connective tissue in the gums. It is not used for anti-inflammatory effects. Minocycline is used to reduce bleeding and pocket depth and to inhibit bacterial growth.

A nurse is teaching a nursing student what is meant by "generations" of cephalosporins. Which statement by the student indicates understanding of the teaching? a. "Cephalosporins are assigned to generations based on their relative costs to administer." b. "Cephalosporins have increased activity against gram-negative bacteria with each generation." c. "First-generation cephalosporins have better penetration of the cerebrospinal fluid." d. "Later generations of cephalosporins have lower resistance to destruction by beta-lactamases."

ANS: B With each progression from first-generation agents to fifth-generation agents, the cephalosporins show increased activity against gram-negative organisms, increased resistance to destruction by beta-lactamases, and increased ability to reach the CSF. Cost is not a definitive factor. First-generation drugs have less penetration of the CSF. Resistance to destruction by beta-lactamases increases with increasing generations.

A 65-year-old female patient tells a nurse that she has begun taking calcium supplements. The nurse learns that the patient consumes two servings of dairy products and takes 1200 mg in calcium supplements each day. The patient's serum calcium level is 11.1 mg/dL. What will the nurse tell this patient? a. She should increase her dietary calcium in addition to the supplements b. The amount of calcium she takes increases her risk for heart attack and stroke c. To continue taking 1200 mg of calcium supplement since she is over age 50 d. To supplement her calcium with 10,000 units of vitamin D each day

ANS: B Women older than 50 years need 1200 mg of calcium per day. Patients should take only enough supplemental calcium to make up for the difference between what the diet provides and the RDA. This patient is getting 1800 mg/day. Excess supplemental calcium can increase risks for vascular calcification, MI, stroke, and kidney stones. She does not need to increase calcium intake. Supplementing with 10,000 units of vitamin D is indicated for documented vitamin D deficiency, which is not evident in this case.

A nursing student is caring for a patient with community-acquired pneumonia (CAP) who also has a methicillin-resistant Staphylococcus aureus (MRSA) skin infection. The prescriber has ordered daptomycin [Cubicin]. The nurse encourages the student to approach the provider to request a different antibiotic because of which facts about daptomycin? (Select all that apply.) a. It causes significant renal impairment. b. It is approved only for bloodstream and skin infections. c. It increases the risk of serious cardiorespiratory events. d. It is more likely to produce resistant strains of bacteria. e. It is not effective against MRSA infections.

ANS: B, C Daptomycin has been approved only for bloodstream and skin infections caused by S. aureus, including MRSA infections, and would not be indicated for a CAP infection. In patients with CAP, daptomycin poses a higher risk of serious cardiorespiratory side effects. Daptomycin does not cause renal impairment and is no more likely to produce bacterial resistance than other antibiotics. Daptomycin can be used to treat MRSA infections.

Which organisms can be treated with penicillin G (Benzylpenicillin)? (Select all that apply.) a. Methicillin-resistant Staphylococcus aureus b. Neisseria meningitidis c. Pseudomonas aeruginosa d. Streptococcus pyogenes e. Treponema pallidum

ANS: B, D, E Penicillin G is the first drug of choice for N. meningitidis. It is a drug of first choice for infections caused by sensitive gram-positive cocci, including S. pyogenes. It is a drug of choice for T. pallidum. It is not effective against methicillin-resistant S. aureus or P. aeruginosa.

A patient has a viral sinus infection, and the provider tells the patient that antibiotics will not be prescribed. The patient wants to take an antibiotic and asks the nurse what possible harm could occur by taking an antibiotic. Which response by the nurse is correct? a. "Antibiotics are mutagenic and can produce changes that cause resistance." b. "Even normal flora can develop resistance and transfer this to pathogens." c. "Host cells become resistant to antibiotics with repeated use." d. "Patients who overuse antibiotics are more likely to have nosocomial infections."

ANS: B- "even normal flora can develop resistance and transfer this to pathogens" Antibiotics make conditions favorable for the overgrowth of microbes with acquired resistance. Normal flora, present at all times, can develop resistance and can transfer this resistance to pathogens if they occur. Even when pathogens are not present, antibiotic use can promote resistance in the future. Antibiotics are not mutagenic. Host cells are not affected. Antibiotic use does not increase the risk of nosocomial infection in a particular patient but does increase resistance in resident organisms in a particular hospital.

An older adult patient with chronic obstructive pulmonary disease (COPD) develops bronchitis. The patient has a temperature of 39.5°C. The nurse will expect the provider to: a. obtain a sputum culture and wait for the results before prescribing an antibiotic. b. order empiric antibiotics while waiting for sputum culture results. c. treat symptomatically, because antibiotics are usually ineffective against bronchitis. d. treat the patient with more than one antibiotic without obtaining cultures.

ANS: B- order empiric antibiotics while waiting for sputum culture results Patients with severe infections should be treated while culture results are pending. If a patient has a severe infection or is at risk of serious sequelae if treatment is not begun immediately, it is not correct to wait for culture results before beginning treatment. Until a bacterial infection is ruled out, treating symptomatically is not indicated. Treating without obtaining cultures is not recommended.

A patient who is taking doxycycline for a serious infection contacts the nurse to report anal itching. The nurse will contact the provider to discuss: a. adding an antihistamine to the patient's drug regimen. b. ordering liver function tests to test for hepatotoxicity. c. prescribing an antifungal drug to treat a superinfection. d. testing the patient for a C. difficile secondary infection.

ANS: C A superinfection occurs secondary to suppression of drug-sensitive organisms. Overgrowth with fungi, especially Candida albicans, is common and may occur in the mouth, pharynx, vagina, and bowel. Anal itching is a sign of such an infection, not a sign of hepatotoxicity. Antihistamines will not treat the cause. C. difficile is characterized by profuse, watery diarrhea.

A child who has juvenile idiopathic arthritis and who has been taking methotrexate [Rheumatrex] will begin a course of abatacept [Orencia]. What will the nurse include when teaching the child's family about this drug? a. That abatacept and methotrexate must both be taken to be effective b. To continue getting vaccinations during therapy with abatacept c. That signs of infection may warrant immediate discontinuation of abatacept d. That a tumor necrosis factor (TNF) antagonist may be added if this therapy is not effective

ANS: C Abatacept suppresses immune function and can increase the risk of serious infection. Parents should report any signs of infection, which may warrant discontinuation of abatacept. Abatacept may be taken alone. Abatacept may blunt the effectiveness of vaccines, and vaccines should be up-to-date prior to therapy and may need to be delayed until 3 months after therapy. Live vaccines should be avoided. Abatacept should not be given with TNF antagonists because of the increased risk of serious infections.

A patient is to begin taking doxycycline to treat a rickettsial infection. Which statement by the patient indicates a need for teaching about this drug? a. "I should consult my provider before using laxatives or antacids while taking this drug." b. "I should not take a calcium supplement or consume dairy products with this drug." c. "I should take this drug with food to ensure more complete absorption." d. "If I get diarrhea, I should stop taking the drug and let my provider know immediately."

ANS: C Absorption of tetracyclines is reduced in the presence of food. The tetracyclines form insoluble chelates with calcium, iron, magnesium, aluminum, and zinc, so patients should not take tetracyclines with dairy products, calcium supplements, or drugs containing these minerals. Patients who experience diarrhea should stop taking the drug and notify the provider so they can be tested for C. difficile infection.

A patient who has developed postmenopausal osteoporosis will begin taking alendronate [Fosamax]. The nurse will teach this patient to take the drug: a. at bedtime to minimize adverse effects. b. for a maximum of 1 to 2 years. c. while sitting upright with plenty of water. d. with coffee or orange juice to increase absorption.

ANS: C Alendronate can cause esophagitis, and this risk can be minimized if the patient takes the drug with water while in an upright position. Taking the drug at bedtime is not indicated. The drug may be taken up to 5 years before re-evaluation is indicated. Coffee and orange juice reduce the absorption of alendronate and should be delayed for 30 minutes after taking the drug.

A patient is about to receive penicillin G for an infection that is highly sensitive to this drug. While obtaining the patient's medication history, the nurse learns that the patient experienced a rash when given amoxicillin [Amoxil] as a child 20 years earlier. What will the nurse do? a. Ask the provider to order a cephalosporin. b. Reassure the patient that allergic responses diminish over time. c. Request an order for a skin test to assess the current risk. d. Suggest using a desensitization schedule to administer the drug.

ANS: C Allergy to penicillin can decrease over time; therefore, in patients with a previous allergic reaction who need to take penicillin, skin tests can be performed to assess the current risk. Until this risk is known, changing to a cephalosporin is not necessary. Reassuring the patient that allergic responses will diminish is not correct, because this is not always the case; the occurrence of a reaction must be confirmed with skin tests. Desensitizing schedules are used when patients are known to be allergic and the drug is required anyway.

A patient is receiving intravenous potassium penicillin G, 2 million units to be administered over 1 hour. At 1900, the nurse notes that the dose hung at 1830 has infused completely. What will the nurse do? a. Assess the skin at the infusion site for signs of tissue necrosis. b. Observe the patient closely for confusion and other neurotoxic effects. c. Request an order for serum electrolytes and cardiac monitoring. d. Watch the patient's actions and report any bizarre behaviors.

ANS: C Although penicillin G is the least toxic of all antibiotics, certain adverse effects may be caused by compounds coadministered with penicillin. When large doses of potassium penicillin G are administered rapidly, hyperkalemia can occur, which can cause fatal dysrhythmias. When penicillin G is administered IM, tissue necrosis occurs with inadvertent intra-arterial injection. Confusion, seizures, and hallucinations can occur if blood levels of the drug are too high. Bizarre behaviors result with large IV doses of procaine penicillin G.

A nurse is reviewing the culture results of a patient receiving an aminoglycoside. The report reveals an anaerobic organism as the cause of infection. What will the nurse do? a. Contact the provider to discuss an increased risk of aminoglycoside toxicity. b. Continue giving the aminoglycoside as ordered. c. Request an order for a different class of antibiotic. d. Suggest adding a penicillin to the patient's drug regimen.

ANS: C Aminoglycosides are not effective against anaerobic microbes, so another class of antibiotics is indicated. There is no associated increase in aminoglycoside toxicity with anaerobic infection. The aminoglycoside will not be effective, so continuing to administer this drug is not indicated. Adding another antibiotic is not useful, because the aminoglycoside is not necessary.

A patient recently began receiving clindamycin [Cleocin] to treat an infection. After 8 days of treatment, the patient reports having 10 to 15 watery stools per day. What will the nurse tell this patient? a. The provider may increase the clindamycin dose to treat this infection. b. This is a known side effect of clindamycin, and the patient should consume extra fluids. c. The patient should stop taking the clindamycin now and contact the provider immediately. d. The patient should try taking Lomotil or a bulk laxative to minimize the diarrheal symptoms.

ANS: C Clostridium difficile-associated diarrhea (CDAD) is the most severe toxicity of clindamycin; if severe diarrhea occurs, the patient should be told to stop taking clindamycin immediately and to contact the provider so that treatment with vancomycin or metronidazole can be initiated. Increasing the dose of clindamycin will not treat this infection. Consuming extra fluids while still taking the clindamycin is not correct, because CDAD can be fatal if not treated. Taking Lomotil or bulk laxatives only slows the transit of the stools and does not treat the cause.

A 6-week-old infant who has not yet received immunizations develops a severe cough. While awaiting nasopharyngeal culture results, the nurse will expect to administer which antibiotic? a. Clindamycin [Cleocin] b. Doxycycline [Vibramycin] c. Erythromycin ethylsuccinate d. Penicillin G

ANS: C Erythromycin is the drug of first choice for infections caused by Bordetella pertussis, the causative agent of whooping cough. Infants who have not received their first set of immunizations are at increased risk of pertussis. Clindamycin, doxycycline, and penicillin are not recommended.

A patient will begin taking etanercept [Enbrel] for severe rheumatoid arthritis. The patient has been taking methotrexate [Rheumatrex]. The patient asks if the etanercept is stronger than the methotrexate. The nurse will tell the patient that etanercept ____ methotrexate. a. has synergistic effects with b. helps reduce adverse effects associated with c. is better at delaying progression of joint damage than d. has fewer adverse effects than

ANS: C Etanercept has been shown to reduce symptoms in patients with moderate to severe RA who have not responded to methotrexate. It does not have synergistic effects with methotrexate or reduce adverse effects of methotrexate. It has many adverse effects.

A nurse provides teaching for a patient with cytomegalovirus (CMV) retinitis who will receive the ganciclovir ocular implant [Vitrasert]. Which statement by the patient indicates a need for further teaching? a. "My vision may be blurred for 2 to 4 weeks after receiving the implant." b. "Surgical placement of the implant is an outpatient procedure." c. "The implant will remain in place permanently." d. "The implant will slow progression of CMV retinitis."

ANS: C Ganciclovir ocular implants must be replaced every 5 to 8 months and do not remain in place permanently. It is correct that vision may be blurred for 2 to 4 weeks after placement of the implant, that placement is an outpatient procedure, and that the implant will slow progression of CMV retinitis.

. A patient with rheumatoid arthritis is taking leflunomide [Arava] and an oral contraceptive. She tells the nurse she would like to get pregnant. What will the nurse tell her? a. That leflunomide is not dangerous during the first trimester of pregnancy b. That plasma levels of leflunomide will drop rapidly when she stops taking it c. To ask her provider about an 11-day course of cholestyramine d. To stop taking leflunomide when she stops using contraception

ANS: C Leflunomide is contraindicated during pregnancy. Patients desiring pregnancy must follow a three-step protocol that includes stopping the drug, taking cholestyramine to chelate the leflunomide, and ensuring that leflunomide drug levels are below 20 mcg/L before getting pregnant. Leflunomide is teratogenic and is not safe during pregnancy. Plasma levels of leflunomide may take 2 years to drop without using cholestyramine. It is not correct to stop taking leflunomide without following the protocol.

A patient who has been taking linezolid [Zyvox] for 6 months develops vision problems. What will the nurse do? a. Reassure the patient that this is a harmless side effect of this drug. b. Tell the patient that blindness is likely to occur with this drug. c. Tell the patient that this symptom is reversible when the drug is discontinued. d. Tell the patient to take tyramine supplements to minimize this effect.

ANS: C Linezolid is associated with neuropathy, including optic neuropathy. This is a reversible effect that will stop when the drug is withdrawn. Reassuring the patient that this is a harmless side effect is not correct. It is not an indication that blindness will occur. Tyramine supplements are not indicated.

A patient shows signs and symptoms of conjunctivitis. Which aminoglycoside would the nurse expect to be ordered? a. Amikacin [Amikin] b. Kanamycin [Kantrex] c. Neomycin [Neomycin] d. Paromomycin [Humatin]

ANS: C Neomycin is used for topical treatment of infections of the eye, ear, and skin. Amikacin, kanamycin, and paromomycin are not topical treatments and are not indicated for eye infections.

A patient who is pregnant has a history of recurrent genital herpesvirus (HSV). The patient asks the nurse what will be done to suppress an outbreak when she is near term. The nurse will tell the patient that: a. antiviral medications are not safe during pregnancy. b. intravenous antiviral agents will be used if an outbreak occurs. c. oral acyclovir [Zovirax] may be used during pregnancy. d. topical acyclovir [Zovirax] must be used to control outbreaks.

ANS: C Oral acyclovir is devoid of serious adverse effects and may be used safely during pregnancy. It is incorrect to tell this patient that antiviral medications are not safe during pregnancy. Oral acyclovir is used to suppress recurrent genital herpes near term; intravenous antiviral medications are not indicated. It is not necessary to rely on topical medications because oral acyclovir is safe.

A patient with an infection caused by Pseudomonas aeruginosa is being treated with piperacillin. The nurse providing care reviews the patient's laboratory reports and notes that the patient's blood urea nitrogen and serum creatinine levels are elevated. The nurse will contact the provider to discuss: a. adding an aminoglycoside. b. changing to penicillin G. c. reducing the dose of piperacillin. d. ordering nafcillin.

ANS: C Patients with renal impairment should receive lower doses of piperacillin than patients with normal renal function. Aminoglycosides are nephrotoxic. Penicillin G and nafcillin are not effective against Pseudomonas infections.

A nursing student asks a nurse why pegylated interferon alfa is used instead of regular interferon for a patient with hepatitis C. The nurse will tell the student that pegylated interferon: a. decreases the need for additional medications. b. has fewer adverse effects than interferon. c. is administered less frequently than interferon. d. may be given orally to increase ease of use.

ANS: C Pegylated interferon alfa preparations are preferred because of their convenience and superior efficacy. These preparations may be given once weekly instead of three or more times per week like the regular interferon. Using pegylated interferons does not decrease the need for additional medications. Pegylated interferons have similar adverse effects. Pegylated interferons are not given orally.

A patient has an infection caused by Streptococcus pyogenes. The prescriber has ordered dicloxacillin PO. What will the nurse do? a. Administer the medication as ordered. b. Contact the provider to suggest giving the drug IV. c. Question the need for a penicillinase-resistant penicillin. d. Suggest ordering vancomycin to treat this infection.

ANS: C Penicillinase-resistant penicillins have been developed for use against penicillinase-producing strains of staphylococci. These drugs have a very narrow antimicrobial spectrum and should be used only for such infections. S. pyogenes can be treated with penicillin G. The nurse should question the order. It is incorrect to contact the provider to ask for IV dosing. This infection can be treated with penicillin G and not with vancomycin.

A postmenopausal patient develops osteoporosis. The patient asks the nurse about medications to treat this condition. The nurse learns that the patient has a family history of breast cancer. The nurse will suggest discussing which medication with the provider? a. Estrogen estradiol b. Pamidronate [Aredia] c. Raloxifene [Evista] d. Teriparatide [Forteo]

ANS: C Raloxifene is a selective estrogen receptor modulator (SERM) that has estrogenic effects in some tissues and antiestrogenic effects in others. It can preserve bone mineral density while protecting against breast and endometrial cancers. Estrogen promotes breast cancer and would not be indicated. Pamidronate and teriparatide are not protective against breast cancer.

A patient has severe Paget's disease of the bone. The patient asks the nurse what can be done to alleviate the pain. The nurse will suggest that the patient discuss the use of which medication with the provider? a. Alendronate [Fosamax] b. Calcifediol [25-Hydroxy-D3] c. Calcitonin-salmon [Miacalcin] d. Long-acting NSAIDs

ANS: C Salmon calcitonin is the drug of choice for rapid relief of pain associated with Paget's disease. Alendronate, calcifediol, and NSAIDs are not indicated.

A patient with second-degree burns is treated with silver sulfadiazine [Silvadene]. A nursing student asks the nurse about the differences between silver sulfadiazine and mafenide [Sulfamylon], because the two are similar products, and both contain sulfonamides. What does the nurse tell the student about silver sulfadiazine? a. It causes increased pain when the medication is applied. b. It has a broader spectrum of antimicrobial sensitivity. c. It has antibacterial effects related to release of free silver. d. It suppresses renal excretion of acid, causing acidosis.

ANS: C Silver sulfadiazine has antibacterial effects primarily related to the release of free silver and not to the sulfonamide portion of the molecule. Unlike mafenide, silver sulfadiazine reduces pain when applied. Silver sulfadiazine and mafenide have similar antibacterial effects. Because silver sulfadiazine does not suppress renal excretion of acid, it has fewer systemic effects.

A patient who is taking immunosuppressant medications develops a urinary tract infection. The causative organism is sensitive to sulfonamides and to another, more expensive antibiotic. The prescriber orders the more expensive antibiotic. The nursing student assigned to this patient asks the nurse why the more expensive antibiotic is being used. Which response by the nurse is correct? a. "Immunosuppressed patients are folate deficient." b. "Patients who are immunosuppressed are more likely to develop resistance." c. "Sulfonamides are bacteriostatic and depend on host immunity to work." d. "Sulfonamides intensify the effects of immunosuppression."

ANS: C Sulfonamides are usually bacteriostatic and require intact host defenses for complete elimination of infection. Immunosuppressed patients are not necessarily folate deficient. There is no increased likelihood of developing bacterial resistance in immunosuppressed patients. Sulfonamides do not affect immunosuppression.

A patient with type 2 diabetes mellitus takes glipizide. The patient develops a urinary tract infection, and the prescriber orders TMP/SMZ. What will the nurse tell the patient? a. Patients with diabetes have an increased risk of an allergic reaction. b. Patients taking TMP/SMZ may need increased doses of glipizide. c. The patient should check the blood glucose level more often while taking TMP/SMZ. d. The patient should stop taking the glipizide while taking the TMP/SMZ.

ANS: C Sulfonamides can intensify the effects of some drugs, including glipizide, which is a sulfonylurea-type hypoglycemic medication. These drugs may require a reduction in dose to prevent toxicity. Patients should monitor their blood glucose more closely. There is no increase in allergic reactions to TMP/SMZ in patients who are diabetic. Patients taking TMP/SMZ may need reduced doses of glipizide but should not stop taking the drug.

To prevent yellow or brown discoloration of teeth in children, tetracyclines should not be given: a. to children once the permanent teeth have developed. b. to patients taking calcium supplements. c. to pregnant patients after the fourth month of gestation. d. with dairy products or antacids.

ANS: C Tetracyclines bind to calcium in developing teeth, resulting in yellow or brown discoloration. They should not be given to pregnant women after the fourth month of gestation, because they will cause staining of deciduous teeth in the fetus. In children, discoloration occurs when tetracyclines are given between the ages of 4 and 8 years, because this is when permanent teeth are developing. Tetracycline binds with calcium, so absorption is diminished when the drug is given with calcium supplements, dairy products, or calcium-containing antacids; however, this does not affect tooth development.

A nurse is providing teaching for a patient with osteoporosis who has just switched from alendronate [Fosamax] to zoledronate [Reclast]. Which statement by the patient indicates a need for further teaching? a. "I will need to have blood tests periodically while taking this drug." b. "I will only need a dose of this medication every 1 to 2 years." c. "This drug is less likely to cause osteonecrosis of the jaw." d. "This drug is only given intravenously."

ANS: C Zoledronate has an increased risk of osteonecrosis of the jaw, as does alendronate. The patient is correct to identify the need for periodic blood tests. Zoledronate is given only every 1 to 2 years and is given only intravenously.

A patient with bronchitis is taking TMP/SMZ, 160/800 mg orally, twice daily. Before administering the third dose, the nurse notes that the patient has a widespread rash, a temperature of 103°F, and a heart rate of 100 beats per minute. The patient looks ill and reports not feeling well. What will the nurse do? a. Administer the dose and request an order for an antipyretic medication. b. Withhold the dose and request an order for an antihistamine to treat the rash. c. Withhold the dose and notify the provider of the symptoms. d. Request an order for intravenous TMP/SMZ, because the patient is getting worse.

ANS: C The most severe hypersensitivity reaction with TMP/SMZ is Stevens-Johnson syndrome, which manifests with fever, malaise, and rash. The drug should be discontinued immediately if a rash occurs. Requesting an antipyretic while giving the drug is incorrect. Giving antihistamines is not indicated. Giving TMP/SMZ intravenously would make the reaction worse.

A nurse is preparing to administer intramuscular penicillin to a patient who is infected with T. pallidum and notes that the order is for sodium penicillin G. Which action is correct? a. Administer the drug as prescribed. b. Contact the provider to discuss administering the drug intravenously. c. Contact the provider to discuss changing the drug to benzathine penicillin G. d. Request an order for piperacillin instead of penicillin G.

ANS: C The procaine and benzathine penicillin salts are absorbed slowly and are considered repository preparations. When benzathine penicillin G is injected IM, penicillin G is absorbed for weeks and is useful only against highly sensitive organisms such as T. pallidum. Sodium penicillin G is absorbed rapidly, with peak effects in 15 minutes. Administering the drug IV will not yield repository effects. Piperacillin is not used for T. pallidum IC

A patient who is receiving a final dose of intravenous (IV) cephalosporin begins to complain of pain and irritation at the infusion site. The nurse observes signs of redness at the IV insertion site and along the vein. What is the nurse's priority action? a. Apply warm packs to the arm, and infuse the medication at a slower rate. b. Continue the infusion while elevating the arm. c. Select an alternate intravenous site and administer the infusion more slowly. d. Request central venous access.

ANS: C These signs indicate thrombophlebitis. The nurse should select an alternative IV site and administer the infusion more slowly. The IV should not be continued in the same site, because necrosis may occur. A central line would be indicated only for long-term administration of antibiotics.

A nurse is caring for a patient who takes an ACE inhibitor and an ARB medication who will begin taking TMP/SMZ to treat a urinary tract infection. Which serum electrolyte will the nurse expect to monitor closely? a. Calcium b. Chloride c. Potassium d. Sodium

ANS: C Trimethoprim suppresses renal excretion of potassium, increasing the risk of hyperkalemia. Patients at greatest risk are those taking high doses of trimethoprim and those taking other drugs that elevate potassium, including ACE inhibitors and ARB medications. Trimethoprim does not affect other serum electrolytes.

An immunocompromised child is exposed to chickenpox and the provider orders valacyclovir [Valtrex] to be given orally three times daily. The nurse will contact the provider to change this order for which reason? a. Valacyclovir is not used as varicella prophylaxis. b. The dosage is too high for this indication. c. The drug may cause serious adverse effects in immunocompromised patients. d. Valacyclovir is not approved for use in children.

ANS: C Valacyclovir is approved for use for varicella in immunocompetent children. In immunocompromised patients, it has produced a syndrome known as thrombotic thrombocytopenic purpura/hemolytic uremia syndrome (TTP/HUS). The dosage is fine for immunocompetent children.

A patient is receiving tobramycin 3 times daily. The provider has ordered a trough level with the 8:00 AM dose. The nurse will ensure that the level is drawn at what time? a. 4:00 AM b. 7:00 AM c. 7:45 AM d. 8:45 AM

ANS: C When a patient is receiving divided doses of an aminoglycoside, the trough level should be drawn just before the next dose; therefore, 7:45 AM would be the appropriate time. It would not be appropriate to draw a trough at the other times listed.

A 55-year-old female patient asks a nurse about calcium supplements. The nurse learns that the patient consumes two servings of dairy products each day. The patient's serum calcium level is 9.5 mg/dL. The serum vitamin D level is 18 ng/mL. The nurse will recommend adding ____ daily and ____ IU of vitamin D3 each day. a. 1200 mg of calcium once; 10,000 b. 1500 mg of calcium twice; 1000 c. 600 mg of calcium once; 10,000 d. 600 mg of calcium twice; 2000

ANS: C Women older than 50 years need 1200 mg of calcium per day. This patient is getting 600 mg/day. She should add 600 mg/day to compensate for what she does not get in her diet, because the amount of a supplement should be enough to make up the difference. Her vitamin D level is low, so she needs a vitamin D supplement. To treat deficiency, adults older than 19 years should get 10,000 IU/day. An additional intake of 1200 mg of calcium once daily is too much calcium. An additional intake of 1500 mg of calcium twice daily is too much calcium, and 1000 IU of vitamin D is not enough to treat deficiency. An additional intake of 600 mg of calcium twice daily is too much calcium, and 2000 IU of vitamin D is not sufficient to treat deficiency.

Which patients should be given antibiotics prophylactically? (Select all that apply.) a. Children who attend day care with other children who have strep throat b. Children undergoing chemotherapy who have mild neutropenia c. Patients with certain congenital heart defects at risk for bacterial endocarditis d. Patients with compound fractures undergoing surgical repair e. Postoperative patients who have undergone emergency cesarean sections

ANS: C, D Antimicrobial prophylaxis is recommended for patients with congenital or other heart disease who have an increased risk for bacterial endocarditis and for patients undergoing certain surgeries in which the risk for infection is very high, including those who have compound fractures. Children who are exposed to strep throat will need to have confirmed strep infection prior to receiving antibiotics. Patients with milk neutropenia do not need antimicrobial prophylaxis. Patients undergoing emergency C-sections need antimicrobial prophylaxis prior to surgery, not after.

Which are benefits of using a combination of two or more antibiotics? (Select all that apply.) a. Reduced cost b. Reduced risk of superinfection c. Reduced toxicity d. Reduced resistance e. Reduced risk in severe infection

ANS: C, D, E In some situations, an antibiotic combination can reduce toxicity, because the dosage of a more toxic agent can be reduced. Drug combinations can be used to suppress the emergence of resistant bacteria in the treatment of tuberculosis. With a severe infection, a combination of antibiotics is useful until culture results are known to ensure that all possible sources of infection are treated. Using multiple antibiotics is more costly and poses a greater risk of superinfection.

A child has received amoxicillin [Amoxil] for three previous ear infections, but a current otitis media episode is not responding to treatment. The nurse caring for this child suspects that resistance to the bacterial agent has occurred by which microbial mechanism? a. Alteration of drug target molecules b. Antagonist production c. Drug inactivation d. Reduction of drug concentration at the site of action

ANS: C- Drug inactivation Drug inactivation can occur when microbes produce drug-metabolizing enzymes. Penicillin-resistant organisms, including many that cause otitis media, produce penicillinase. Alteration of drug target molecules, drug inactivation, and reduction of the drug concentration occur with other antimicrobials.

A nursing student asks a nurse to clarify the differences between the mechanisms of spontaneous mutation and conjugation in acquired resistance of microbes. What will the nurse say? a. Conjugation results in a gradual increase in resistance. b. Conjugation results in random changes in the microbe's DNA. c. Spontaneous mutation leads to resistance to only one antimicrobial agent. d. Spontaneous mutation can transfer DNA from one organism to another.

ANS: C- Spontaneous mutation leads to resistance only one antimicrobial agent Spontaneous mutation generally confers resistance to only one drug. Conjugation can occur quickly; spontaneous mutation is gradual. Spontaneous mutation is random; conjugation is not. Conjugation can occur with the transfer of DNA from one organism to another.

A recent campaign, initiated by the Centers for Disease Control (CDC), to delay the emergence of antibiotic resistance in hospitals, has what as one of its objectives? a. Allowing patients to stop antibiotics when symptoms subside b. Allowing prescribers to develop their own prescribing guidelines c. Increased adherence to prescribed antibiotics d. Increased use of antibiotics among parents of young children

ANS: C- increased adherence to prescribed antibiotics The CDC initiative has identified three objectives to help delay the emergence of resistance to antibiotics in hospitals. One of the objectives is to increase adherence to prescribed antibiotics among users. Allowing patients to stop using antibiotics before the prescription ends is not one of the objectives and may increase the risk of resistance. Other objectives include ensuring that prescribers adhere to appropriate prescribing guidelines instead of developing their own and decreasing antibiotic use among parents of small children.

A patient is taking alendronate [Fosamax] to treat Paget's disease. The patient asks the nurse why calcium supplements are necessary. The nurse will tell the patient that calcium supplements are necessary to: a. reduce the likelihood of atrial fibrillation. b. maximize bone resorption of calcium. c. minimize the risk of esophageal cancer. d. prevent hyperparathyroidism.

ANS: D Alendronate can induce hyperparathyroidism in patients with Paget's disease; calcium supplementation can prevent this effect. Giving calcium does not reduce the incidence of atrial fibrillation, maximize bone resorption of calcium, or minimize the risk of esophageal cancer.

A patient is being treated with warfarin [Coumadin] to prevent thrombus. The patient develops hyperuricemia, and the provider orders allopurinol [Zyloprim]. The nurse will contact the provider to discuss ____ the ____ dose. a. increasing; allopurinol b. increasing; warfarin c. reducing; allopurinol d. reducing; warfarin

ANS: D Allopurinol can inhibit hepatic drug-metabolizing enzymes and thus delay the inactivation of other drugs. This is a particular concern in patients taking warfarin; therefore, the warfarin dose should be reduced when allopurinol is also used. It is not correct to increase the allopurinol dose, increase the warfarin dose, or reduce the allopurinol dose.

The parent of an infant with otitis media asks the nurse why the prescriber has ordered amoxicillin [Amoxil] and not ampicillin [Unasyn]. What will the nurse tell the parent? a. Amoxicillin is a broader spectrum antibiotic than ampicillin. b. Amoxicillin is not inactivated by beta-lactamases. c. Ampicillin is associated with more allergic reactions. d. Ampicillin is not as acid stable as amoxicillin.

ANS: D Amoxicillin and ampicillin are similar in structure and actions but differ primarily in acid stability. Amoxicillin is more acid stable and, when administered orally, results in higher blood levels than can be obtained with equivalent doses of ampicillin. The two drugs have the same spectrum, both are inactivated by beta-lactamases, and both can cause allergic reactions.

A patient who has gout will begin taking febuxostat [Uloric] and colchicine. What will the nurse include when teaching this patient about this drug regimen? a. "You are taking both drugs in order to prevent hepatic side effects." b. "You may stop taking the febuxostat after your uric acid levels decrease." c. "You will have to take both drugs indefinitely to treat your symptoms." d. "You will stop taking the colchicine within 6 months after starting therapy."

ANS: D At the beginning of therapy with fevuxostat, symptoms of gout may flare, so colchicine or NSAIDS are given for up to 6 months to alleviate this. The combination does not prevent effects on the liver. The febuxostat will be given indefinitely.

A child with otitis media has had three ear infections in the past year. The child has just completed a 10-day course of amoxicillin [Amoxil] with no improvement. The parent asks the nurse why this drug is not working, because it has worked in the past. What will the nurse tell the patient? a. "Amoxicillin is too narrow in spectrum." b. "The bacteria have developed a three-layer cell envelope." c. "The bacteria have developed penicillin-binding proteins (PBPs) that have a low affinity for penicillins." d. "The bacteria have synthesized penicillinase."

ANS: D Beta-lactamases are enzymes that cleave the beta-lactam ring and render the PCN inactive. This resistance is common with organisms that cause ear infections. Amoxicillin is a broad-spectrum antibiotic. A three-layer cell envelope occurs in gram-negative bacteria. Some bacterial strains, including methicillin-resistant Staphylococcus aureus (MRSA), develop PBPs with a low affinity for penicillins. MRSA is not a common cause of otitis media.

A patient who is taking calcium supplements receives a prescription for ciprofloxacin [Cipro] for a urinary tract infection. The nurse will teach this patient to: a. consume extra fluids while taking the ciprofloxacin to prevent hypercalciuria. b. stop taking the calcium supplements while taking the ciprofloxacin. c. take the two medications together to increase the absorption of both. d. take the calcium either 6 hours before or 2 hours after taking the ciprofloxacin.

ANS: D Cationic compounds, including calcium supplements, can reduce the absorption of ciprofloxacin, so proper interval dosing is necessary. Consuming extra fluids is not indicated. With proper interval dosing, it is not necessary to discontinue the calcium while giving the ciprofloxacin. These two medications should not be given together.

A patient who has cystic fibrosis has a Pseudomonas aeruginosa infection and the provider has ordered aztreonam [Cayston]. What will the nurse teach this patient about administration of this drug? a. Administer the drug intramuscularly twice daily. b. Give a daily dose every day for 28 days and then stop. c. Inhale the powdered drug as ordered three times each day. d. Use the nebulizer to administer the drug three times daily.

ANS: D Cayston is a form of aztreonam formulated for inhalation administration for patients with cystic fibrosis who have P. aeruginosa lung infections. The reconstituted powder is given using a nebulizer system three times daily for 28 days followed by 28 days off. This form of the drug is not given IM. The dose is three times daily. The drug is reconstituted and administered via a nebulizer.

Which cephalosporin may be used to treat meningitis? a. Cefaclor b. Cefazolin c. Cefoxitin d. Cefotaxime

ANS: D Cefotaxime has increased ability to reach the cerebrospinal fluid (CSF) and to treat meningitis. Cefaclor, cefazolin, and cefoxitin do not reach effective concentrations in the CSF.

A patient has a skin infection and the culture reveals methicillin-resistant Staphylococcus aureus (MRSA). What is an appropriate treatment for this patient? a. Cefaclor b. Cefazolin c. Cefotaxime d. Ceftaroline

ANS: D Ceftaroline is a fifth-generation cephalosporin with a spectrum similar to third-generation cephalosporins but also with activity against MRSA. Cefaclor is a second-generation cephalosporin. Cefazolin is a first-generation cephalosporin. Cefotaxime is a third-generation cephalosporin.

The nurse is caring for a patient who will begin receiving intravenous ciprofloxacin [Cipro] to treat pyelonephritis. The nurse learns that the patient has a history of myasthenia gravis. Which action by the nurse is correct? a. Administer the ciprofloxacin and monitor the patient for signs of muscle weakness. b. Ask the provider whether the ciprofloxacin can be given orally. c. Request an order for concurrent administration of metronidazole [Flagyl]. d. Suggest that the provider order a different antibiotic for this patient.

ANS: D Ciprofloxacin and other fluoroquinolones can exacerbate muscle weakness in patients with myasthenia gravis and should not be given to these patients. It is not correct to administer the drug and monitor for this effect. Giving the drug by a different route will not alter this effect. Metronidazole is given when C. difficile occurs.

A patient with gout who has increasingly frequent acute gouty attacks will begin receiving allopurinol [Zyloprim] and colchicine. The nurse will include which statement when teaching the patient about this drug regimen? a. "Allopurinol helps reduce the gastrointestinal side effects of colchicine." b. "Allopurinol reduces the likelihood of gouty episodes that usually occur with initial colchicine therapy." c. "The colchicine is given to enhance the effects of the allopurinol." d. "You will take both drugs initially and then stop taking the colchicine."

ANS: D Colchicine is used for prophylaxis when urate-lowering drugs, such as allopurinol, are initiated, because gouty episodes have a tendency to increase during this time. Patients start with both drugs, and ultimately the colchicine is withdrawn. Allopurinol does not affect the GI side effects caused by colchicine. Allopurinol may precipitate an acute gouty attack when treatment is begun; colchicine is given to prevent a gouty episode. Colchicine does not enhance the effects of allopurinol.

A hospitalized patient who is taking demeclocycline [Declomycin] reports increased urination, fatigue, and thirst. What will the nurse do? a. Contact the provider to report potential toxic side effects. b. Notify the provider to discuss changing the medication to doxycycline. c. Perform bedside glucometer testing to evaluate the serum glucose level. d. Provide extra fluids and reassure the patient that these are expected side effects.

ANS: D Demeclocycline stimulates urine flow and sometimes is used to treat patients with syndrome of inappropriate antidiuretic hormone secretion (SIADH). The patient should be reassured that increased urination, fatigue, and thirst are known side effects of demeclocycline. It is not correct to notify the provider of toxic side effects or to request another tetracycline. Glucometer testing is not necessary, because the increased urination is not related to an elevated blood glucose level.

A child has been receiving chloramphenicol for a Neisseria meningitidis central nervous system (CNS) infection. The nurse administers the dose and subsequently notes that the child has vomited and appears dusky and gray in color. The child's abdomen is distended. What will the nurse do? a. Contact the provider for an order to obtain a chloramphenicol level. b. Notify the provider that the child's meningitis is worsening. c. Recognize this as initial signs of a C. difficile infection. d. Stop the infusion immediately and notify the provider.

ANS: D Gray syndrome is a potentially fatal toxicity associated with chloramphenicol use. When symptoms occur, the drug should be stopped immediately. Lower chloramphenicol levels may prevent gray syndrome, but lowering the dose will not stop symptoms once they have appeared. These are not signs of worsening meningitis or a C. difficile infection.

A postmenopausal patient is at high risk for developing osteoporosis. The patient's prescriber orders raloxifene [Evista], and the nurse provides teaching about this drug. Which statement by the patient indicates understanding of the teaching? a. "I may experience breast tenderness while taking this drug." b. "I may experience fewer hot flashes while taking this drug." c. "I should discontinue this drug several weeks before any surgery." d. "I should walk as much as possible during long airline flights."

ANS: D Like estrogen, raloxifene increases the risk of deep vein thrombosis. Patients taking this drug should be cautioned to take walks on long flights or whenever they must sit for long periods. The drug does not increase breast tenderness or decrease hot flashes. There is no need to discontinue this drug before surgery.

A nurse teaches a patient about sulfonamides. Which statement by the patient indicates a need for further teaching? a. "I need to drink extra fluids while taking this medication." b. "I need to use sunscreen when taking this drug." c. "I should call my provider if I develop a rash while taking this drug." d. "I should stop taking this drug when my symptoms are gone."

ANS: D Patients should always be advised to complete the prescribed course of the antibiotic even when symptoms subside. Patients should also understand the need to drink 8 to 10 glasses of water a day, to use sunscreen, and to notify the provider if they develop a rash.

A patient with chronic gout has an acute gouty episode and is admitted to the hospital. The patient has been taking nonsteroidal anti-inflammatory drugs for several months. The prescriber plans to begin therapy with probenecid. What will the nurse do? a. Give the medication as ordered and observe the patient closely for gastrointestinal side effects. b. Request an order to lower the dose of the nonsteroidal antiinflammatory drug. c. Restrict the patient's fluid intake to minimize the risk of renal injury. d. Suggest delaying the probenecid therapy until the acute episode has subsided.

ANS: D Probenecid may exacerbate acute episodes of gout, so treatment with this drug should be delayed until the acute attack has passed. Probenecid has mild GI effects. Lowering the dose of the NSAID is not recommended. Patients should increase their fluid intake to minimize the risk of renal injury.

A patient about to begin therapy with etanercept has a positive tuberculin skin test. A chest radiograph is negative. The nurse will expect this patient to: a. begin taking antituberculosis drugs at the beginning of treatment with etanercept. b. have periodic chest radiographs during treatment with etanercept. c. have regular monitoring of symptoms to detect active tuberculosis. d. undergo tuberculosis treatment prior to beginning etanercept treatment.

ANS: D Since tuberculosis (TB) in a patient taking etanercept is often extrapulmonary and disseminated, it is important to test all patients for TB. Those who test positive for latent TB should be treated for TB before etanercept treatment is begun. It is not correct to begin TB treatment concurrently with etanercept treatment. Latent TB must be treated and not monitored.

A nurse is caring for an African American patient who has been admitted to the unit for long-term antibiotic therapy with sulfonamides. The patient develops fever, pallor, and jaundice. The nurse would be correct to suspect that the patient has developed: a. Stevens-Johnson syndrome. b. kernicterus. c. hepatotoxicity. d. hemolytic anemia.

ANS: D Sulfonamides can cause hemolytic anemia in patients of African American and Mediterranean origin, usually because of a genetic deficiency. Red cell lysis can produce fever, pallor, and jaundice, and patients should be observed for these signs. The patient's signs and symptoms are not characteristic of Stevens-Johnson syndrome or hepatotoxicity. The patient's signs and symptoms are not characteristic of kernicterus, which occurs in newborns

A nurse preparing to administer intravenous gentamicin to a patient notes that the dose is half the usual dose for an adult. The nurse suspects that this is because this patient has a history of: a. antibiotic resistance. b. interpatient variation. c. liver disease. d. renal disease.

ANS: D The aminoglycosides are eliminated primarily by the kidneys, so in patients with renal disease, doses should be reduced or the dosing interval should be increased to prevent toxicity. Patients with antibiotic resistance would be given amikacin. Interpatient variation may occur but cannot be known without knowing current drug levels. Aminoglycosides are not metabolized by the liver, so liver disease would not affect drug levels.

A nurse is discussing microbial resistance among sulfonamides and trimethoprim with a nursing student. Which statement by the student indicates a need for further teaching? a. "Bacterial resistance to trimethoprim is relatively uncommon." b. "Resistance among gonococci, streptococci, and meningococci to sulfonamides is high." c. "Resistance to both agents can occur by spontaneous mutation of organisms." d. "Resistance to sulfonamides is less than resistance to trimethoprim."

ANS: D There is less microbial resistance to trimethoprim than there is to sulfonamides. Bacterial resistance to trimethoprim is relatively uncommon. Gonococcal, streptococcal, and meningococcal resistance to sulfonamides is especially high. For both agents, resistance can develop by spontaneous mutation.

A patient develops CDAD. Which antibiotic is recommended for treating this infection? a. Chloramphenicol b. Clindamycin [Cleocin] c. Linezolid [Zyvox] d. Vancomycin

ANS: D Vancomycin and metronidazole are the drugs of choice for treating CDAD.

A patient reports experiencing weakness, fatigue, nausea, vomiting, constipation, and nocturia. Total serum calcium is 10.5 mg/dL. A dipstick urinalysis shows a positive result for protein. When questioned, the patient reports taking vitamin D and calcium supplements. The nurse will counsel the patient to: a. reduce the amount of vitamin D and stop taking the calcium. b. discuss taking calcitonin-salmon [Fortical] with the provider. c. stop both supplements and discuss the use of a diuretic with the provider. d. stop taking vitamin D, reduce the amount of calcium, and increase the fluid intake.

ANS: D Vitamin D toxicity can occur, and early responses include the symptoms described. Patients should be counseled to stop taking vitamin D, reduce their calcium intake, and increase their fluid intake. It is not correct to reduce the vitamin D intake and the calcium intake. Calcitonin-salmon is not indicated. A diuretic is indicated when hypercalciuria is severe.

A patient with recurrent bacterial pneumonia is treated with an antibiotic that has worked previously but is not working to reduce symptoms in a current infection. The nurse caring for this patient understands that this is likely for which reason? a. The antibiotic altered the genetic makeup of the bacterial strain causing this infection. b. The antibiotic caused host cells to change and become more susceptible to bacterial effects. c. The antibiotic caused a mutation of the organism leading to reduced drug sensitivity. d. The antibiotic destroyed competing organisms that secrete substances toxic to the pathogen.

ANS: D - the antibiotic destroyed competing organisms that secrete substances toxic to the pathogen Antibiotics promote resistance by destroying sensitive organisms and eliminating the toxins they produce, which facilitates survival of resistant organisms. Antibiotics do not alter the genetic makeup of bacteria. The host cells are not affected. Antibiotics are not mutagenic

The nurse has been caring for a patient who has been taking antibiotics for 3 weeks. Upon assessing the patient, the nurse notices the individual has developed oral thrush. What describes the etiology of the thrush? a. Antibiotic resistance b. Community-acquired infection c. Nosocomial infection d. Superinfection

ANS: D- Superinfection Oral thrush is a manifestation of a superinfection. The development of thrush is not a symptom of antibiotic resistance. Oral thrush typically is not a community-acquired infection. The development of thrush is not a nosocomial infection.

The nurse is caring for a patient on a medical-surgical unit who has a fever of unknown origin. The prescriber has ordered a broad-spectrum antibiotic. Which intervention is the priority? a. Administering the antibiotic immediately b. Administering antipyretics as soon as possible c. Delaying administration of the antibiotic until the culture results are available d. Obtaining all cultures before the antibiotic is administered

ANS: D. Obtaining all cultures before the antibiotic is administered It is essential that samples of exudates and body fluids (in this case, blood cultures) be obtained for culture before initiation of treatment. Administration of the antibiotic is important but not more important than obtaining specimens for culture. Antipyretics may be indicated, but the priority is obtaining specimens for culture. Treatment may be initiated before the test results are available.

A parent asks a nurse if the provider will prescribe an antibiotic for a child who attends school with several children who have strep throat. The child is complaining of a sore throat and has a fever. What will be the nurse's response? a. "Because strep throat is likely, your child should be treated empirically." b. "With good hand washing, your child should not get strep throat." c. "Your child probably has strep throat, so your provider will order an antibiotic." d. "Your child should come to the clinic to have a throat culture done today."

ANS: D: "Your child should come to the clinic to have a throat culture done today" Whenever possible, the infecting organism should be identified before antibiotics are started, even if there is a strong suspicion that a particular organism is present. The nurse is correct in telling the parent to bring the child to the clinic for a throat culture. Fever and sore throat have other causes, so it is not correct to treat this child empirically, especially because these symptoms are not severe or life threatening. Although good hand washing is always indicated, this child already has symptoms and needs to be evaluated. Antibiotics should not be started until indicated by cultures.

A patient has a localized skin infection, which is most likely caused by a gram-positive cocci. Until the culture and sensitivity results are available, the nurse will expect the provider to order a ____-spectrum ____ agent. a. broad; systemic b. broad; topical c. narrow; systemic d. narrow; topical

ANS: D; narrow; topical When infections are treated before the causative agent has been identified, and after cultures have been obtained, antibiotics may be used based on the knowledge of which microbes are most likely to cause infection at that particular site. Because this is a localized infection, a topical agent is recommended. Unless the infection is very serious, a narrow-spectrum antibiotic is best.

The nurse is teaching a nursing student about the mechanism by which antimicrobial agents achieve selective toxicity. Which statement by the student indicates a need for further teaching? a. "Some agents disrupt the bacterial cell wall." b. "Some agents act to block the conversion of para-aminobenzoic acid (PABA) to folic acid." c. "Some agents cause phagocytosis of bacterial cells." d. "Some agents weaken the cell wall, causing cell wall lysis."

ANS:C-- some agents cause phagocytosis of bacterial cells Antimicrobial agents do not cause phagocytosis of bacterial cells. They do disrupt and weaken the bacterial cell wall; because human cells do not have cell walls, antimicrobial agents are not toxic to human cells. Some agents block the conversion of PABA to folic acid; humans do not synthesize folic acid and are not harmed by this process.

Which fluoroquinolone antibiotics may be administered to children? (Select all that apply.) a. Ciprofloxacin [Cipro] b. Levofloxacin [Levaquin] c. Moxifloxacin [Avelox] d. Norfloxacin [Noroxin] e. Ofloxacin

Ciprofloxacin and levofloxacin are the only two fluoroquinolones approved for use in children.


Conjuntos de estudio relacionados

Unidad 1 Capítulo 1 — "La enajenación y la asimilación"

View Set

Life 1st time through - Score 77.5%

View Set

All of the following are secondary succesion except

View Set

A&P Laboratory 9: Skeletal System: Articulations/Joints

View Set